Как решать выражения с корнями: Выражения с квадратными корнями — урок. Алгебра, 8 класс.

Содержание

Как решать примеры с корнями

Корнем n степени из числа называют такое число, которое при возведении в эту степень даст то число, из которого извлекается корень. Чаще всего, действия производятся с корнями квадратными, которые соответствуют 2 степени. При извлечении корня часто невозможно найти его явно, а результатом является число, которое невозможно представить в виде натуральной дроби (трансцендентное). Но используя некоторые приемы, можно значительно упростить решение примеров с корнями.Вам понадобится

Если не требуется абсолютная точность, при решении примеров с корнями воспользуйтесь калькулятором. Чтобы извлечь из числа квадратный корень, наберите его на клавиатуре, и просто нажмите соответствующую кнопку, на которой изображен знак корня. Как правило, на калькуляторах берется корень квадратный. Но для вычисления корней высших степеней, воспользуйтесь функцией возведения числа в степень (на инженерном калькуляторе).

Для извлечения квадратного корня возведите число в степень 1/2, кубического корня в 1/3 и так далее. 4=(-2)∙ (-2)∙ (-2)∙ (-2)=16. Для извлечения квадратного корня нацело, когда это возможно, воспользуйтесь таблицей квадратов натуральных чисел.

Если же рядом нет калькулятора, или требуется абсолютная точность в расчетах, используйте свойства корней, а также различные формулы для упрощения выражений. Из многих чисел можно извлечь корень частично. Для этого воспользуйтесь свойством, что корень из произведения двух чисел равен произведению корней из этих чисел √m∙n=√m∙√n.

Пример. Вычислите значение выражения (√80-√45)/ √5. Прямое вычисление ничего не даст, поскольку нацело не извлекается ни один корень. Преобразуйте выражение (√16∙5-√9∙5)/ √5=(√16∙√5-√9∙√5)/ √5=√5∙(√16-√9)/ √5. Произведите сокращение числителя и знаменателя на √5, получите (√16-√9)=4-3=1.

Если подкоренное выражение или сам корень возведены в степень, то при извлечении корня воспользуйтесь тем свойством, что показатель степени подкоренного выражения можно поделить на степень корня. Если деление производится нацело, число вносится из-под корня. 4=5²=25.

Пример. Вычислить значение выражения (√3+√5)∙(√3-√5). Примените формулу разности квадратов и получите (√3)²-(√5)²=3-5=-2.

Вычислить значение выражения с корнями

Умение работать с числовыми выражениями, содержащими квадратный корень, необходимо для успешного решения ряда задач из ОГЭ и ЕГЭ. Как правило, на этих экзаменах достаточно базового представления о том, что такое извлечение корня и как оно осуществляется на практике.

Определение

Корень степени n из числа X – это такое число x, для которого верно равенство: x n = X.

Найти значение выражения с корнем – это значит найти x при известных X и n.

Квадратный корень или, что то же самое, корень второй степени из X – число x, для которого выполнено равенство: x 2 = X.

Обозначение: ∛Х. Здесь 3 – степень корня, Х – подкоренное выражение. Знак ‘√’ часто называют радикалом.

Если над корнем не стоит число, указывающее на степень, то по умолчанию подразумевается степень 2.

В школьном курсе для четных степеней обычно не рассматривают отрицательные корни и подкоренные выражения. Например, не существует √-2, а для выражения √4 верным ответом считается 2, несмотря на то, что (-2) 2 тоже равняется 4.

Рациональность и иррациональность корней

Наиболее простое из возможных заданий c корнем – найти значение выражения либо проверить его на рациональность.

Например, вычислить значения √25; ∛8; ∛-125:

  • √25 = 5, так как 5 2 = 25;
  • ∛8 = 2, так как 2 3 = 8;
  • ∛ – 125 = -5, так как (-5) 3 = -125.

Ответы в приведенных примерах – это рациональные числа.

При работе с выражениями, не содержащими буквенных констант и переменных, рекомендуется всегда выполнять подобную проверку с помощью обратной операции возведения в натуральную степень. Нахождение числа x в n-й степени эквивалентно вычислению произведения n множителей x.

Существует множество выражений с корнем, значение которых иррационально, то есть записывается в виде бесконечной непериодической дроби.

По определению рациональные – это те, что можно выразить обыкновенной дробью, а иррациональные – все остальные действительные числа.

К таким относятся √24, √0,1, √101.

Если в задачнике сказано: найдите значение выражения с корнем из 2, 3, 5, 6, 7 и т. д., то есть из тех натуральных чисел, которые не содержатся в таблице квадратов, то в правильном ответе √2 может присутствовать (когда не оговорено обратное).

Проведение оценки

В задачах с открытым ответом, если найти значение выражения с корнем и записать его рациональным числом невозможно, результат следует оставить в виде радикала.

Некоторые задания могут потребовать проведения оценки. Например, сравнить 6 и √37. Для решения требуется возвести оба числа в квадрат и сравнить результаты. Из двух чисел больше то, чей квадрат больше. Данное правило работает для всех положительных чисел:

Точно так же решаются задачи, в которых несколько чисел надо расставить в порядке возрастания или убывания. 2 = 4 cdot 7 = 28) .

Найдите значение выражения (frac <sqrt<12>cdot sqrt<540>><sqrt<30>>) .

Перепишем исходное выражение, занеся все числа под один корень:

Разложим выражение под корнем на множители так, чтобы среди них были полные квадраты:

(sqrt <216>= sqrt <4 cdot 9 cdot 6>= 2 cdot 3 cdot sqrt <6>= 6sqrt<6>) .

Найдите значение выражения (frac <sqrt<150>cdot sqrt<216>><sqrt<90>>) .

Перепишем исходное выражение, занеся все числа под один корень:

Разложим выражение под корнем на множители так, чтобы среди них были полные квадраты:

Найдите значение выражения (4sqrt <3>cdot sqrt <2>cdot 4sqrt<6>) .

Преобразуем (sqrt <6>= sqrt <2>cdot sqrt<3>) .

Найдем произведение множителей без корня, а множители с корнем сгруппируем:

(4sqrt <3>cdot sqrt <2>cdot 4 sqrt <2>cdot sqrt <3>=16 cdot sqrt<3>^2 cdot sqrt<2>^2 = 16 cdot 3 cdot 2 = 96) .

Этот видеоурок доступен по абонементу

У вас уже есть абонемент? Войти

Повторение свойств квадратных корней

Вкратце повторим теорию и напомним основные свойства квадратных корней.

Свойства квадратных корней:

1. 2. ;

3. ;

4. .

Примеры на упрощение выражений с корнями

Перейдем к примерам использования этих свойств.

Пример 1. Упростить выражение .

Решение. Для упрощения число 120 необходимо разложить на простые множители:

. Квадрат суммы раскроем по соответствующей формуле:

.

Пример 2. Упростить выражение .

Решение. Учтем, что данное выражение имеет смысл не при всех возможных значениях переменной, т. к. в данном выражении присутствуют квадратные корни и дроби, что приводит к «сужению» области допустимых значений. ОДЗ: Ответ. Пример 3. Упростить выражение .

Решение. Видно, что вторая скобка числителя имеет неудобный вид и нуждается в упрощении, попробуем разложить ее на множители с помощью метода группировки.

. Для возможности выносить общий множитель мы упростили корни путем их разложения на множители. Подставим полученное выражение в исходную дробь:

. После сокращения дроби применяем формулу разности квадратов.

Пример на избавление от иррациональности

Пример 4. Освободиться от иррациональности (корней) в знаменателе: а)

Читайте также:  Как войти в биос acer swift

.

.

Ответ.Пример на доказательство и на выделение полного квадрата в сложном радикале

Пример 5. Докажите равенство .

Доказательство. Воспользуемся определением квадратного корня, из которого следует, что квадрат правого выражения должен быть равен подкоренному выражению:

, получили верное равенство.

Пример 6. Упростить выражение .

Решение. Указанное выражение принято называть сложным радикалом (корень под корнем). В данном примере необходимо догадаться выделить полный квадрат из подкоренного выражения. Для этого заметим, что из двух слагаемых . Подставим это выражение под корень:

Ответ..

На этом занятии мы заканчиваем тему «Функция . Свойства квадратного корня», а на следующем уроке начинаем новую тему «Действительные числа».

1. Башмаков М.И. Алгебра 8 класс. – М.: Просвещение, 2004.

2. Дорофеев Г.В., Суворова С.Б., Бунимович Е.А. и др. Алгебра 8. – 5-е изд. – М.: Просвещение, 2010.

3. Никольский С.М., Потапов М.А., Решетников Н.Н., Шевкин А.В. Алгебра 8 класс. Учебник для общеобразовательных учреждений. – М.: Просвещение, 2006.

Дополнительные рекомендованные ссылки на ресурсы сети Интернет

1. Интернет-портал xenoid.ru (Источник).

2. Математическая школа (Источник).

3. Интернет-портал XReferat.Ru (Источник).

1. №357, 360, 372, 373, 382. Дорофеев Г.В., Суворова С.Б., Бунимович Е.А. и др. Алгебра 8. – 5-е изд. – М.: Просвещение, 2010.

2. Избавьтесь от иррациональности в знаменателе: а) 3. Упростите выражение: а) 4. Докажите тождество .

Если вы нашли ошибку или неработающую ссылку, пожалуйста, сообщите нам – сделайте свой вклад в развитие проекта.

Область определения выражения под корнем

Среди всех иррациональных уравнений и неравенств, содержащих корень в знаменателе дроби, можно, в первую очередь, выделить те, которые содержат и , или подобные иррациональные выражения. Эти иррациональные уравнения аналогичны рациональным уравнениям из §II.3.

Задача заключается в выделении полного квадрата выражения . И перехода к квадратному уравнению с новой переменной.

Затем нужно выделить те уравнения и неравенства, где всю иррациональную дробь можно заменить на новую переменную, например, в уравнении:

Можно ввести замену и получить уравнение:t + 2/t = 3

Еще можно выделить уравнения и неравенства, в которых можно преобразовать дроби, содержащие корень, за счет различных алгебраических преобразований, среди которых наиболее популярна разность квадратов и нередко встречаются делители 1.

За счет этих преобразований нередко удается избавиться от иррациональности в знаменателе. Например уравнение:

За счет использования формулы разности квадратов в числителе дроби превращается в:

Если к уравнению или неравенству не удается применить ни один из описанных выше подходов, тогда, чаще всего, решение следует начинать с приведения всех слагаемых к общему знаменателю.

Причем если знаменатель содержит только корни, то его можно отбросить даже при решении неравенств, так как он является положительным

. Например в неравенстве:

После приведения к общему знаменателю получим знаменатель , который можно отбросить и решать далее неравенство:

В некоторых простых случаях, особенно когда имеется одна дробь под знаком корня, предпочтительнее сначала возвести в квадрат, а потом приводить к общему знаменателю. (Приведение к общему знаменателю должно сопровождаться его выделением из под корня, что есть не вполне корректная операция. )

Внимание ловушка! Будьте внимательны при расчете ОДЗ уравнений и неравенств, содержащих дроби и корни:

  1. ОДЗ корня в знаменателе задается неравенством “Подкоренное выражение > 0”, а не “Подкоренное выражение ³ 0”, например в неравенстве:
    ОДЗ будет x О ( 0 ; 6 ), а не x О [ 0 ; 6 ].
  2. ОДЗ дроби под корнем, , отличается от ОДЗ отношения корней, , в первом случае ОДЗ задается системой неравенства во втором – системойЭто объясняется тем, что если числитель и знаменатель отрицательны, то сама дробь положительна и корень извлечь из нее можно, но при этом ни из числителя, ни из знаменателя корень извлечь нельзя. (И это несмотря на то, что при преобразовании выражений эти два выражения обычно считаются эквивалентными.) Например, ОДЗ для выражения будет , а для ОДЗ для будет
    x
    О ( 0 ; Ґ ).
  3. Отбрасывая знаменатель не забывайте, что он все таки влияет на ОДЗ.

Внимательными нужно быть и при расчете ДУ, возникающего при возведении в квадрат дробей и произведений многочленов.

Арифметический квадратный корень. Мини-курс. Уроки 17 — 24. — Math

Преобразование выражений с корнем. Приведение дробей с квадратным корнем к общему знаменателю. Иррациональные дроби. Арифметический квадратный корень. Упростить выражение с квадратным корнем. Деление дробей с корнем. Сокращение дробей с корнем. Умножение выражений с квадратным корнем. Разложение на множители выражений с корнем. Разложение на множители выражений с иррациональностью. Алгебра 8 класс. Примеры с решением. Задания с объяснением. Иррациональные выражения. Выражения с иррациональностью. Математика. Образование.

 

 

Урок 18. Найти значение выражения. Арифметический квадратный корень.

Найти значение выражения. Арифметический квадратный корень. Выделение полного квадрата в выражениях с корнем. Алгебра 8 класс. Иррациональные выражения. Выражения с корнем. Примеры с решением. Преобразовать выражение с корнем. Дробь с корнем. Примеры с корнем. Найти значение выражения с корнем. Привести выражение с корнем к общему знаменателю. Радикал. Примеры с радикалами. Значение радикала. Преобразование выыражений с радикалами. Математика. Образование.

 

 

 Урок 19. Упростить выражение и найти его значение. Квадратный корень.

Вычисление значений арифметического квадратного корня. Когда ставить модуль при извлечении квадратного корня? Как правильно раскрыть модуль. Свойства арифметического квадратного корня. Определение модуля. Примеры с решением. Алгебра 8 класс. Математика. Образование.

  • Пример 1: Найти значение выражения.
  • Пример 2: Упростить выражение и найти его значение.

 

 

 Урок 20. Вычисление значений квадратного корня при помощи формул сокращенного умножения.

Вычисление значений арифметического квадратного корня при помощи формул сокращенного умножения. Как выделить полный квадрат в подкоренном умножении? Как выделить полный квадрат в выражении с корнем. Формулы сокращенного умножения для вычисления значений квадратного корня. Примеры с решением. Алгебра 8 класс. Математика. Образование.

  • Пример 1: Найти значение выражения, разложив подкоренное выражение на множители.
  • Пример 2: Упростить выражение с арифметическим квадратным корнем, выделив полный квадрат в подкоренном выражении.

 

 

 Урок 21. Упростить выражение, выделив полный квадрат под корнем.

Как выделить полный квадрат в подкоренном умножении? Как выделить полный квадрат в выражении с корнем. Формулы сокращенного умножения для вычисления значений квадратного корня. Корень в корне. Корень под корнем. Как вычислить корень под корнем. Как раскрыть модуль при вычислении арифметического квадратного корня? Свойства корня. Примеры с решением. Алгебра 8 класс. Математика. Образование.

  • Пример 1: Найти значение выражения, выделив полный квадрат в подкоренном выражении.
  • Пример 2: Упростить выражение с арифметическим квадратным корнем, выделив полный квадрат в подкоренном выражении.

 

 

 Урок 22. Упростить выражение с корнем. Найти значение корня. Задания с *.  

Как выделить полный квадрат в подкоренном умножении? Как выделить полный квадрат в выражении с корнем. Формулы сокращенного умножения для вычисления значений квадратного корня. Как раскрыть модуль в выражении с корнем. Корень в корне. Корень под корнем. Как вычислить корень под корнем. Как раскрыть модуль при вычислении арифметического квадратного корня? Свойства корня. Примеры с решением. Алгебра 8 класс. Математика. Образование.

  • Пример 1: Найти значение выражения, преобразовав подкоренное выражение и раскрыв модуль.
  • Пример 2: Упростить выражение с арифметическим квадратным корнем, выделив полный квадрат в подкоренном выражении и раскрыв затем модуль.
  • Пример 3: Известна сумма корней. Найти их произведение.

 

 

 Урок 23. Нахождение приблизительного значения квадратного корня.

 Два способа нахождения приблизительного значения арифметического квадратного корня. Алгебра 8 класс. Арифметический квадратный корень. Примеры с решением. Математика. Образование.

 

 

Урок 24. Ветвь параболы. Построение графика. Нахождение значений.

График функции «у» равно корень из «x». Ветвь параболы. Построение графика корня. Нахождение значений по графику. Алгебра 8 класс. Примеры с решением:

Пример 1: Дана функция. Найдите:

  • 1) значение функции, если значение аргумента равно 4; 5;
  • 2) значение аргумента, при котором значение функции равно 2; 2,5.

Пример 2: Не выполняя построения графика функции, укажите, через какие из данных точек проходит этот график.

Пример 3: Постройте в одной системе координат графики функций и укажите координаты точки их пересечения.

Пример 4: Не выполняя построения, найдите координаты точки пересечения графика функции у = V* и прямой.

Пример 5: Постройте график функции. Пользуясь графиком, найдите:

  • 1) значение функции, если значение аргумента равно 9; 7;
  • 2) значение аргумента, при котором значение функции равно 2; 2,5.

 

Начало курса. Арифметический квадратный корень. Уроки 1 — 5.

Мини-курс. Рациональные дроби. Алгебра 7-8 класс.

 Арифметический квадратный корень. Ответы к заданиям из видео уроков.

Преобразования иррациональных выражений | О математике понятно

Иррациональные выражения и их преобразования

        В прошлый раз мы вспомнили (или узнали — кому как), что же такое корень n-й степени, научились извлекать такие корни, разобрали по винтикам основные свойства корней и решали несложные примеры с корнями.

        Этот урок будет продолжением предыдущего и будет посвящён преобразованиям самых разных выражений, содержащих всевозможные корни. Такие выражения называются иррациональными. Здесь появятся и выражения с буквами, и дополнительные условия, и избавление от иррациональности в дробях, и некоторые продвинутые приёмы в работе с корнями. Те приёмы, которые будут рассматриваться в данном уроке, станут хорошей базой для решения задач ЕГЭ (и не только) практически любого уровня сложности. Итак, давайте приступим.

        Прежде всего я продублирую здесь основные формулы и свойства корней. Чтобы не скакать из темы в тему. Вот они:

 при 

        Формулы эти надо обязательно знать и уметь применять. Причём в обе стороны — как слева направо, так и справа налево. Именно на них и основывается решение большинства заданий с корнями любой степени сложности. Начнём пока с самого простого — с прямого применения формул или их комбинаций.

Простое применение формул

        В этой части будут рассматриваться простые и безобидные примеры — без букв, дополнительных условий и прочих хитростей. Однако даже в них, как правило, имеются варианты. И чем навороченнее пример, тем больше таких вариантов. И у неопытного ученика возникает главная проблема — с чего начинать? Ответ здесь простой — не знаешь, что нужно — делай что можно. Лишь бы ваши действия шли в мире и согласии с правилами математики и не противоречили им.) Например, такое задание:

        Вычислить: 

        Даже в таком простеньком примере возможны несколько путей к ответу.

        Первый — просто перемножить корни по первому свойству и извлечь корень из результата:

        Второй вариант такой:  не трогаем, работаем с . Выносим множитель из-под знака корня, а дальше — по первому свойству. Вот так:

        Решать можно как больше нравится. В любом из вариантов ответ получается один — восьмёрка. Мне, например, проще перемножить 4 и 128 и получить 512, а из этого числа отлично извлекается кубический корень. Если кто-то не помнит, что 512 — это 8 в кубе, то не беда: можно записать 512 как 29 (первые 10 степеней двойки, я надеюсь, помните?) и по формуле корня из степени:

        Другой пример.

        Вычислить: .

        Если работать по первому свойству (всё загнать под один корень), то получится здоровенное число, из которого корень потом извлекать — тоже не сахар. Да и не факт, что он извлечётся ровно.) Поэтому здесь полезно в числе  вынести множители из-под корня. Причём вынести по максимуму:

        И теперь всё наладилось:

        Осталось восьмёрку и двойку записать под одним корнем (по первому свойству) и — готово дело. 🙂

        Добавим теперь немного дробей.

        Вычислить:

        

        Пример совсем примитивный, однако и в нём имеются варианты. Можно с помощью вынесения множителя преобразовать числитель и сократить со знаменателем:

        А можно сразу воспользоваться формулой деления корней:

        Как видим, и так, и сяк — всяко правильно.) Если не споткнуться на полпути и не ошибиться. Хотя где тут ошибаться-то…

        Разберём теперь самый последний пример из домашнего задания прошлого урока:

        Упростить:

            

        Совершенно немыслимый набор корней, да ещё и вложенных. Как быть? Главное — не бояться! Здесь мы первым делом замечаем под корнями числа 2, 4 и 32 — степени двойки. Первое что нужно сделать — привести все числа к двойкам: всё-таки чем больше одинаковых чисел в примере и меньше разных, тем проще.) Начнём отдельно с первого множителя:

        Число  можно упростить, сократив двойку под корнем с четвёркой в показателе корня:

        Теперь, согласно корню из произведения:

.

        В числе  выносим двойку за знак корня: 

        А с выражением  расправляемся по формуле корня из корня:

        Значит, первый множитель запишется вот так:

.

        Вложенные корни исчезли, числа стали поменьше, что уже радует. Вот только корни разные, но пока так и оставим. Надо будет — преобразуем к одинаковым. Берёмся за второй множитель.)

        Второй множитель преобразовываем аналогично, по формуле корня из произведения и корня из корня. Где надо — сокращаем показатели по пятой формуле:

        Вставляем всё в исходный пример и получаем:

        Получили произведение целой кучи совершенно разных корней. Неплохо было бы привести их все к одному показателю, а там — видно будет. Что ж, это вполне возможно. Наибольший из показателей корней равен 12, а все остальные — 2, 3, 4, 6 — делители числа 12. Поэтому будем приводить все корни по пятому свойству к одному показателю — к 12:

        Считаем и получаем:

        Красивого числа не получили, ну и ладно. Нас просили упростить выражение, а не посчитать. Упростили? Конечно! А вид ответа (целое число или нет) здесь уже не играет никакой роли.

Немного сложения / вычитания и формул сокращённого умножения

        К сожалению, общих формул для сложения и вычитания корней в математике нету. Однако, в заданиях сплошь и рядом встречаются эти действия с корнями. Здесь необходимо понимать, что любые корни — это точно такие же математические значки, как и буквы в алгебре.) И к корням применимы те же самые приёмы и правила, что и к буквам — раскрытие скобок, приведение подобных, формулы сокращённого умножения и т.п.

        Например, каждому ясно, что . Точно так же одинаковые корни можно совершенно спокойно между собой складывать/вычитать:

        Если корни разные, то ищем способ сделать их одинаковыми — внесением/вынесением множителя или же по пятому свойству. Если ну никак не упрощается, то, возможно, преобразования более хитрые.

        Смотрим первый пример.

        Найти значение выражения: .

        Все три корня хоть и кубические, но из разных чисел. Чисто не извлекаются и между собой складываются/вычитаются. Стало быть, применение общих формул здесь не катит. Как быть? А вынесем-ка множители в каждом корне. Хуже в любом случае не будет.) Тем более что других вариантов, собственно, и нету:

        Стало быть, .

        Вот и всё решение. Здесь мы от разных корней перешли к одинаковым с помощью вынесения множителя из-под корня. А затем просто привели подобные.) Решаем дальше.

        Найти значение выражения

        С корнем из семнадцати точно ничего не поделаешь. Работаем по первому свойству — делаем из произведения двух корней один корень:

        А теперь присмотримся повнимательнее. Что у нас под большим кубическим корнем? Разность ква.. Ну, конечно! Разность квадратов:

        Теперь осталось только извлечь корень: .

        Дальше очень похожий пример, но посложнее.

        Вычислить: 

        Здесь придётся проявить математическую смекалку.) Мыслим примерно следующим образом: «Так, в примере произведение корней. Под одним корнем разность, а под другим — сумма. Очень похоже на формулу разности квадратов. Но… Корни — разные! Первый квадратный, а второй — четвёртой степени… Хорошо бы сделать их одинаковыми. По пятому свойству можно легко из квадратного корня сделать корень четвёртой степени. Для этого достаточно подкоренное выражение возвести в квадрат.»

        Если вы мыслили примерно так же, то вы — на полпути к успеху. Совершенно верно! Превратим первый множитель в корень четвёртой степени. Вот так:

       Теперь, ничего не поделать, но придётся вспомнить формулу квадрата разности. Только в применении к корням. Ну и что? Чем корни хуже других чисел или выражений?! Возводим:

       «Хм, ну возвели и что? Хрен редьки не слаще. Стоп! А если вынести четвёрку под корнем? Тогда выплывет то же самое выражение, что и под вторым корнем, только с минусом, а ведь именно этого мы и добиваемся!»

        Верно! Выносим четвёрку:

.

        А теперь — дело техники:

.

        Вот так распутываются сложные примеры. ) Теперь пора потренироваться с дробями.

        Вычислить:

        

        Ясно, что надо преобразовывать числитель. Как? По формуле квадрата суммы, разумеется. У нас есть ещё варианты разве? 🙂 Возводим в квадрат, выносим множители, сокращаем показатели (где надо):

        Во как! Получили в точности знаменатель нашей дроби. ) Значит, вся дробь, очевидно, равна единице:

        Ещё пример. Только теперь на другую формулу сокращённого умножения.)

        Вычислить:

            

        Понятно, что квадрат разности надо в дело применять. Выписываем знаменатель отдельно и — поехали!

        Выносим множители из-под корней:

        Следовательно, 

.

        Теперь всё нехорошее великолепно сокращается и получается:

        Что ж, поднимаемся на следующий уровень. 🙂

Буквы и дополнительные условия

        Буквенные выражения с корнями — штука более хитрая, чем числовые выражения, и является неиссякаемым источником досадных и очень грубых ошибок. Перекроем этот источник.) Ошибки всплывают из-за того, что частенько таких заданиях фигурируют отрицательные числа и выражения. Они либо даны нам прямо в задании, либо спрятаны в буквах и дополнительных условиях. А нам в процессе работы с корнями постоянно надо помнить, что в корнях чётной степени как под самим корнем, так и в результате извлечения корня должно быть неотрицательное выражение. Ключевой формулой в задачах этого пункта будет четвёртая формула:

        С корнями нечётной степени вопросов никаких — там всегда всё извлекается что с плюсом, что с минусом. И минус, если что, выносится вперёд. Будем сразу разбираться с корнями чётных степеней.) Например, такое коротенькое задание.

        Упростить: , если .

        Казалось бы, всё просто. Получится просто икс. ) Но зачем же тогда дополнительное условие  ? В таких случаях полезно прикинуть на числах. Чисто для себя.) Если , то икс — заведомо отрицательное число. Минус три, например. Или минус сорок. Пусть . Можно минус три возвести в четвёртую степень? Конечно! Получится 81. Можно из 81 извлечь корень четвёртой степени? А почему нет? Можно! Получится тройка. Теперь проанализируем всю нашу цепочку:

        Что мы видим? На входе было отрицательное число, а на выходе — уже положительное. Было минус три, стало плюс три.) Возвращаемся к буквам. Вне всяких сомнений, по модулю это будет точно икс, но только сам икс у нас с минусом (по условию!), а результат извлечения (в силу арифметического корня!) должен быть с плюсом. Как получить плюс? Очень просто! Для этого достаточно перед заведомо отрицательным числом поставить минус. ) И правильное решение выглядит так:

        Кстати сказать, если бы мы воспользовались формулой , то, вспомнив определение модуля, сразу получили бы верный ответ. Поскольку

|x| = -x при x<0.

        Дальше тренируемся.)

        Вынести множитель за знак корня: , где .

        Первый взгляд — на подкоренное выражение. Тут всё ОК. При любом раскладе оно будет неотрицательным. Начинаем извлекать. По формуле корня из произведения, извлекаем корень из каждого множителя:

        Откуда взялись модули, объяснять, думаю, уже не надо.) А теперь анализируем каждый из модулей.

        Множитель |a| так и оставляем без изменений: у нас нету никакого условия на букву a. Мы не знаем, положительное она или отрицательная. Следующий модуль |b2| можно смело опустить: в любом случае выражение b2 неотрицательно. А вот насчёт |c3| — тут уже задачка. ) Если , то и c3<0. Стало быть, модуль надо раскрыть с минусом: |c3| = —c3. Итого верное решение будет такое:

        А теперь — обратная задача. Не самая простая, сразу предупреждаю!

        Внести множитель под знак корня: .

        Если вы сразу запишете решение вот так

,

то вы попали в ловушку. Это неверное решение! В чём же дело?

        Давайте вглядимся в выражение под корнем . Под корнем четвёртой степени, как мы знаем, должно находиться неотрицательное выражение. Иначе корень смысла не имеет.) Поэтому  А это, в свою очередь, значит, что  и, следовательно, само  также неположительно: .

        И ошибка здесь состоит в том, что мы вносим под корень неположительное число : четвёртая степень превращает его в неотрицательное и получается неверный результат — слева заведомый минус, а справа уже плюс. А вносить под корень чётной степени мы имеем право только неотрицательные числа или выражения. А минус, если есть, оставлять перед корнем.) Как же нам выделить неотрицательный множитель в числе , зная, что оно само стопудово отрицательное? Да точно так же! Поставить минус.) А чтобы ничего не поменялось, скомпенсировать его ещё одним минусом. Вот так:

        И теперь уже неотрицательное число (-b) спокойно вносим под корень по всем правилам:

        Этот пример наглядно показывает, что, в отличие от других разделов математики, в корнях правильный ответ далеко не всегда вытекает автоматически из формул. Необходимо подумать и лично принять верное решение.) Особенно следует быть внимательнее со знаками в иррациональных уравнениях и неравенствах.

        Разбираемся со следующим важным приёмом в работе с корнями — избавлением  от иррациональности.

Избавление от иррациональности в дробях

        Если в выражении присутствуют корни, то, напомню, такое выражение называется выражением с иррациональностью. В некоторых случаях бывает полезно от этой самой иррациональности (т.е. корней) избавиться. Как можно ликвидировать корень? Корень у нас пропадает при… возведении в степень. С показателем либо равным показателю корня, либо кратным ему. Но, если мы возведём корень в степень (т.е. помножим корень сам на себя нужное число раз), то выражение от этого поменяется. Нехорошо.) Однако в математике бывают темы, где умножение вполне себе безболезненно. В дробях, к примеру. Согласно основному свойству дроби, если числитель и знаменатель умножить (разделить) на одно и то же число, то значение дроби не изменится.

        Допустим, нам дана вот такая дробь:

        Можно ли избавиться от корня в знаменателе? Можно! Для этого корень надо возвести в куб. Чего нам не хватает в знаменателе для полного куба? Нам не хватает множителя , т.е. . Вот и домножаем числитель и знаменатель дроби на 

        Корень в знаменателе исчез. Но… он появился в числителе. Ничего не поделать, такова судьба. ) Нам это уже не важно: нас просили знаменатель от корней освободить. Освободили? Безусловно.)

        Кстати, те, кто уже в ладах с тригонометрией, возможно, обращали внимание на то, что в некоторых учебниках и таблицах, к примеру,  обозначают по-разному: где-то , а где-то . Вопрос — что правильно? Ответ: всё правильно! ) Если догадаться, что  – это просто результат освобождения от иррациональности в знаменателе дроби . 🙂

        Зачем нам освобождаться от иррациональности в дробях? Какая разница — в числителе корень сидит или в знаменателе? Калькулятор всё равно всё посчитает.) Ну, для тех, кто не расстаётся с калькулятором, разницы действительно практически никакой… Но, даже считая на калькуляторе, можно обратить внимание на то, что делить на целое число всегда удобнее и быстрее, чем на иррациональное. А уж про деление в столбик вообще умолчу.)

        Следующий пример только подтвердит мои слова.

        Освободиться от иррациональности в знаменателе дроби:

        Как здесь ликвидировать квадратный корень в знаменателе? Если числитель и знаменатель помножить на выражение , то в знаменателе получится квадрат суммы. Сумма квадратов первого и второго чисел дадут нам просто числа безо всяких корней, что очень радует. Однако… всплывёт удвоенное произведение первого числа на второе, где корень из трёх всё равно останется. Не канает. Как быть? Вспомнить другую замечательную формулу сокращённого умножения! Где никаких удвоенных произведений, а только квадраты:

        Такое выражение, которое при домножении какой-то суммы (или разности) выводит на разность квадратов, ещё называют сопряжённым выражением. В нашем примере сопряжённым выражением будет служить разность . Вот и домножаем на эту разность числитель и знаменатель:

        Что тут можно сказать? В результате наших манипуляций не то что корень из знаменателя исчез — вообще дробь исчезла! 🙂 Даже с калькулятором отнять корень из трёх от тройки проще, чем считать дробь с корнем в знаменателе. Ещё пример.

        Освободиться от иррациональности в знаменателе дроби:

        Как здесь выкручиваться? Формулы сокращённого умножения с квадратами сразу не катят — не получится полной ликвидации корней из-за того, что корень у нас в этот раз не квадратный, а кубический. Надо, чтобы корень как-то возвёлся в куб. Стало быть, применять надо какую-то из формул с кубами. Какую? Давайте подумаем. В знаменателе — сумма . Как нам добиться возведения корня в куб? Домножить на неполный квадрат разности! Значит, применять будем формулу суммы кубов. Вот эту:

        В качестве a у нас тройка, а в качестве b — корень кубический из пяти:

        И снова дробь исчезла.) Такие ситуации, когда при освобождении от иррациональности в знаменателе дроби у нас вместе с корнями полностью исчезает сама дробь, встречаются очень часто. Как вам вот такой примерчик!

        Вычислить:

        

        Попробуйте просто сложить эти три дроби! Без ошибок! 🙂 Один общий знаменатель чего стоит. А что, если попробовать освободиться от иррациональности в знаменателе каждой дроби? Что ж, пробуем:

        Ух ты, как интересно! Все дроби пропали! Напрочь. И теперь пример решается в два счёта:

           

        Просто и элегантно. И без долгих и утомительных вычислений. 🙂

        Именно поэтому операцию освобождения от иррациональности в дробях надо уметь делать. В подобных навороченных примерах только она и спасает, да.) Разумеется, внимательность никто не отменял. Бывают задания, где просят избавиться от иррациональности в числителе. Эти задания ничем от рассмотренных не отличаются, только от корней очищается числитель.)

Более сложные примеры

        Осталось рассмотреть некоторые специальные приёмы в работе с корнями и потренироваться распутывать не самые простые примеры. И тогда полученной информации уже будет достаточно для решения заданий с корнями любого уровня сложности. Итак — вперёд.) Для начала разберёмся, что делать со вложенными корнями, когда формула корня из корня не работает. Например, вот такой примерчик.

        Вычислить: 

        Корень под корнем… К тому же под корнями сумма или разность. Стало быть, формула корня из корня (с перемножением показателей) здесь не действует. Значит, надо что-то делать с подкоренными выражениями: у нас просто нету других вариантов. В таких примерах чаще всего под большим корнем зашифрован полный квадрат какой-нибудь суммы. Или разности. А корень из квадрата уже отлично извлекается! И теперь наша задача — его расшифровать.) Такая расшифровка красиво делается через систему уравнений. Сейчас всё сами увидите.)

        Итак, под первым корнем у нас вот такое выражение:

        А вдруг, не угадали? Проверим! Возводим в квадрат по формуле квадрата суммы:

        Всё верно.) Но… Откуда я взял это выражение ? С неба?

        Нет.) Мы его чуть ниже получим честно. Просто по данному выражению я показываю, как именно составители заданий шифруют такие квадраты. 🙂 Что такое 54? Это сумма квадратов первого и второго чисел. Причём, обратите внимание, уже без корней! А корень остаётся в удвоенном произведении, которое в нашем случае равно . Поэтому распутывание подобных примеров начинается с поиска удвоенного произведения. Если распутывать обычным подбором. И, кстати, о знаках. Тут всё просто. Если перед удвоенным плюс, то квадрат суммы. Если минус, то разности.) У нас плюс — значит, квадрат суммы.) А теперь — обещанный аналитический способ расшифровки. Через систему.)

        Итак, у нас под корнем явно тусуется выражение (a+b)2, и наша задача — найти a и b. В нашем случае сумма квадратов даёт 54. Вот и пишем:

        Теперь удвоенное произведение. Оно у нас . Так и записываем:

        Получили вот такую системку:

        Решаем обычным методом подстановки. Выражаем из второго уравнения, например,  и подставляем в первое:

        Решим первое уравнение:

           

           

       Получили биквадратное уравнение относительно a. Считаем дискриминант: 

       Значит,

       Получили аж четыре возможных значения a. Не пугаемся. Сейчас мы всё лишнее отсеем.) Если мы сейчас для каждого из четырёх найденных значений  посчитаем соответствующие значения , то получим четыре решения нашей системы. Вот они:

        И тут вопрос — а какое из решений нам подходит? Давайте подумаем. Отрицательные решения можно сразу отбросить: при возведении в квадрат минусы «сгорят», и всё подкоренное выражение в целом не изменится.) Остаются первые два варианта. Выбрать их можно совершенно произвольно: от перестановки слагаемых сумма всё равно не меняется.) Пусть, например, , а .

        Итого получили под корнем квадрат вот такой суммы:

        Всё чётко.)

        Я не зря так детально описываю ход решения. Чтобы было понятно, как происходит расшифровка.) Но есть одна проблемка. Аналитический способ расшифровки хоть и надёжный, но весьма длинный и громоздкий: приходится решать биквадратное уравнение, получать четыре решения системы и потом ещё думать, какие из них выбрать… Хлопотно? Согласен, хлопотно. Этот способ безотказно работает в большинстве подобных примеров. Однако очень часто можно здорово сократить себе работу и найти оба числа творчески. Подбором.) Да-да! Сейчас, на примере второго слагаемого (второго корня), я покажу более лёгкий и быстрый способ выделения полного квадрата под корнем.

        Итак, теперь у нас вот такой корень: .

        Размышляем так: «Под корнем — скорее всего, зашифрованный полный квадрат. Раз перед удвоенным минус — значит, квадрат разности. Сумма квадратов первого и второго чисел даёт нам число 54. Но какие это квадраты? 1 и 53? 49 и 5? Слишком много вариантов… Нет, лучше начать распутывать с удвоенного произведения. Наши  можно расписать как  . Раз произведение удвоенное, то двойку сразу отметаем. Тогда кандидатами на роль a и b остаются 7 и . А вдруг, это 14 и /2? Не исключено. Но начинаем-то всегда с простого!» Итак, пусть , а . Проверим их на сумму квадратов:

        Получилось! Значит, наше подкоренное выражение — это на самом деле квадрат разности:

 

        Вот такой вот способ-лайт, чтобы не связываться с системой. Не всегда работает, но во многих таких примерах его вполне достаточно. Итак, под корнями — полные квадраты. Осталось только правильно извлечь корни, да досчитать пример:

        А теперь разберём ещё более нестандартное задание на корни.)

        Докажите, что число A – целое, если .

        Впрямую ничего не извлекается, корни вложенные, да ещё и разных степеней… Кошмар! Однако, задание имеет смысл.) Стало быть, ключ к его решению имеется.) А ключ здесь такой. Рассмотрим наше равенство

как уравнение относительно A. Да-да! Хорошо бы избавиться от корней. Корни у нас кубические, поэтому возведём-ка обе части равенства в куб. По формуле куба суммы:

       Кубы и корни кубические друг друга компенсируют, а под каждым большим корнем забираем одну скобку у квадрата и сворачиваем произведение разности и суммы в разность квадратов:

        Отдельно сосчитаем разность квадратов под корнями:

        Отлично! Значит, всё наше равенство ещё сильнее упростится:

   

        А теперь делаем финт ушами — заменяем сумму корней в скобках на A (согласно условию примера!).

        Получаем кубическое уравнение  или .

        Здесь как раз тот случай, когда один из корней легко угадывается — это . Значит, наш многочлен  можно разложить как

        Как разложить? Либо по схеме Горнера, либо делением «уголком» на скобку (A-4), либо даже группировкой (если представить -3A как -16A+13A). Объяснять подробно деление уголком или схему Горнера в теме про корни — уже совсем отклоняться от курса.) Кто в теме — и так поймёт.

        А теперь легко заметить, что квадратный трёхчлен во вторых скобках имеет отрицательный дискриминант, а значит, наше уравнение имеет единственный действительный корень . И поэтому наша страшная сумма корней  в действительности равна просто 4. То есть, явно целому числу. Что и требовалось доказать.)

        А теперь — поупрощаем некоторые дробные выражения с корнями. От простого — к сложному. Здесь всё точно так же, как и с многочленами. Только в применении к корням. ) Я же говорил, что действия с корнями ничем не отличаются от таковых с буквами. И к корням с таким же успехом применима вся алгебра седьмого класса — формулы сокращённого умножения, разложение на множители, приведение подобных и т.п.

        Например, такое задание.

        Сократить дробь:

        

        Пример явно намекает на применение формулы разности квадратов:

        Спрашивается, а где же здесь квадраты? Сплошные корни… Сейчас покажу. 🙂

        Берём числитель нашей дробушки: .

        Что такое ? По свойству корня из степени, мы можем вынести квадрат наружу. Вот так:

        Хорошо, а из  как квадрат сделать? Не вопрос! По пятому свойству, домножаем на двойку показатели корня и подкоренного выражения:

        По такой технологии, между прочим, можно совершенно любой корень превратить в совершенно любую степень. Какую хотим. 🙂  Как, например,  представить в виде 4-й степени? Нет проблем: 

        Хотим из степеней корни делаем, хотим — наоборот, степени из корней. Что хотим, то и творим. Математика, однако! 🙂

        Итак, весь наш числитель можно представить как разность квадратов:

        А дальше никаких проблем — раскладываем числитель на множители и сокращаем:

        Следующий пример.

        Упростить:

        

        Действуем аналогично. Раскладываем на множители и сокращаем. 🙂 В числителе применяем группировку. Например, вот такую:

        А в знаменателе просто выносим общий множитель :

        Подставляем всё в нашу дробь и сокращаем:

        Как видим, разложение на множители очень популярно в теме с корнями. Очень! И особенно — формула разности квадратов. Именно поэтому формулы сокращённого умножения так важно знать и уметь применять. 🙂

        Ну и на десерт распутаем что-нибудь навороченное. )

        Упростить:

        

        Чтобы не запутаться и не наляпать ошибок, будем действовать по порядку. При взгляде на любой пример всегда задаём сами себе вопрос: «Что в примере мне больше всего не нравится?» В данном примере большинство скажет: «Числитель первой дроби!» Верно! Вот и упростим его отдельно: остальная часть примера от этого никак не пострадает.) Итак,

        Вместо знака деления удобно использовать черту дроби. Вот так:

        Сначала упростим дробь. Как? Попробуем сократить.) Для этого, ясное дело, надо разложить на множители числитель и знаменатель, да… Берём отдельно числитель . Можно его разложить на множители? Можно! Для этого из a надо сделать корень. Вот так:

        Если теперь подставить вместо a выражение , то всплывёт общий множитель. 🙂

        Со знаменателем полная аналогия:

        Таким образом,

        Теперь от упрощённой дроби отнимаем единичку. Как? Делаем из единички дробь и — вперёд!

        Следующим пунктом идёт деление полученной дроби на выражение . Это означает, что оно пойдёт у нас в знаменатель:

        Уфф… Дальше… Отнимаем от полученного выражения дробь :

        И, наконец, последнее усилие. Возводим результат в куб:

        Ну как, всё понятно? Тогда — вперёд, набиваем руку и делаем примеры!

        Вычислить:

        Вынести множители за знак корня: ,  , где .

        Внести множители под знак корня: ,  .

        Освободиться от иррациональности в знаменателе дробей:

        , .

        Вычислить: 

        Доказать, что A – целое число, если .

        Упростить:

        Ответы (пока) давать не буду — иначе неинтересно. 🙂 До встречи и успехов!

Сложение корней с разными степенями. Сложение и вычитание корней, примеры. Что такое квадратный корень

Квадратным корнем из числа X называется число A , которое в процессе умножения самого на себя (A * A ) может дать число X .
Т.е. A * A = A 2 = X , и √X = A .

Над квадратными корнями (√x ), как и над другими числами, можно выполнять такие арифметические операции, как вычитание и сложение. Для вычитания и сложения корней их нужно соединить посредством знаков, соответствующих этим действиям (например √x — √y ).
А потом привести корни к их простейшей форме — если между ними окажутся подобные, необходимо сделать приведение. Оно заключается в том, что берутся коэффициенты подобных членов со знаками соответствующих членов, далее заключаются в скобки и выводится общий корень за скобками множителя. Коэффициент, который мы получили, упрощается по обычным правилам.

Шаг 1. Извлечение квадратных корней

Во-первых, для сложения квадратных корней сначала нужно эти корни извлечь. Это можно будет сделать в том случае, если числа под знаком корня будут полными квадратами. Для примера возьмем заданное выражение √4 + √9 . Первое число 4 является квадратом числа 2 . Второе число 9 является квадратом числа 3 . Таким образом, можно получить следующее равенство: √4 + √9 = 2 + 3 = 5 .
Все, пример решен. Но так просто бывает далеко не всегда.

Шаг 2. Вынесение множителя числа из-под корня

Если полных квадратов нет под знаком корня, можно попробовать вынести множитель числа из-под знака корня. Для примера возьмём выражение √24 + √54 .

Раскладываем числа на множители:
24 = 2 * 2 * 2 * 3 ,
54 = 2 * 3 * 3 * 3 .

В числе 24 мы имеем множитель 4 , его можно вынести из-под знака квадратного корня. В числе 54 мы имеем множитель 9 .

Получаем равенство:
√24 + √54 = √(4 * 6) + √(9 * 6) = 2 * √6 + 3 * √6 = 5 * √6 .

Рассматривая данный пример, мы получаем вынос множителя из-под знака корня, тем самым упрощая заданное выражение.

Шаг 3. Сокращение знаменателя

Рассмотрим следующую ситуацию: сумма двух квадратных корней — это знаменатель дроби, например, A / (√a + √b) .
Теперь перед нами стоит задача «избавиться от иррациональности в знаменателе».
Воспользуемся следующим способом: умножаем числитель и знаменатель дроби на выражение √a — √b .

Формулу сокращённого умножения мы теперь получаем в знаменателе:
(√a + √b) * (√a — √b) = a — b .

Аналогично, если в знаменателе имеется разность корней: √a — √b , числитель и знаменатель дроби умножаем на выражение √a + √b .

Возьмём для примера дробь:
4 / (√3 + √5) = 4 * (√3 — √5) / ((√3 + √5) * (√3 — √5)) = 4 * (√3 — √5) / (-2) = 2 * (√5 — √3) .

Пример сложного сокращения знаменателя

Теперь будем рассматривать достаточно сложный пример избавления от иррациональности в знаменателе.

Для примера берём дробь: 12 / (√2 + √3 + √5) .
Нужно взять её числитель и знаменатель и перемножить на выражение √2 + √3 — √5 .

Получаем:

12 / (√2 + √3 + √5) = 12 * (√2 + √3 — √5) / (2 * √6) = 2 * √3 + 3 * √2 — √30.

Шаг 4. Вычисление приблизительного значения на калькуляторе

Если вам требуется только приблизительное значение, это можно сделать на калькуляторе путём подсчёта значения квадратных корней. Отдельно для каждого числа вычисляется значение и записывается с необходимой точностью, которая определяется количеством знаков после запятой. Далее совершаются все требуемые операции, как с обычными числами.

Пример вычисления приблизительного значения

Необходимо вычислить приблизительное значение данного выражения √7 + √5 .

В итоге получаем:

√7 + √5 ≈ 2,65 + 2,24 = 4,89 .

Обратите внимание: ни при каких условиях не следует производить сложение квадратных корней, как простых чисел, это совершенно недопустимо. То есть, если сложить квадратный корень из пяти и из трёх, у нас не может получиться квадратный корень из восьми.

Полезный совет: если вы решили разложить число на множители, для того, чтобы вывести квадрат из-под знака корня, вам необходимо сделать обратную проверку, то есть перемножить все множители, которые получились в результате вычислений, и в конечном результате этого математического расчёта должно получиться число, которое нам было задано первоначально.

В наше время современных электронных вычислительных машин вычисление корня из числа не представляется сложной задачей. Например, √2704=52, это вам подсчитает любой калькулятор. К счастью, калькулятор есть не только в Windows, но и в обычном, даже самом простеньком, телефоне. Правда если вдруг (с малой долей вероятности, вычисление которой, между прочим, включает в себя сложение корней) вы окажитесь без доступных средств, то, увы, придется рассчитывать только на свои мозги.

Тренировка ума никогда не помещает. Особенно для тех, кто не так часто работает с цифрами, а уж тем более с корнями. Сложение и вычитание корней — хорошая разминка для скучающего ума. А еще я покажу поэтапно сложение корней. Примеры выражений могут быть следующие.

Уравнение, которое нужно упростить:

√2+3√48-4×√27+√128

Это иррациональное выражение. Для того чтобы его упростить нужно привести все подкоренные выражения к общему виду. Делаем поэтапно:

Первое число упростить уже нельзя. 2×2)

Переписываем выражение с упрощенными слагаемыми:

√2+12×√3-12×√3+8×√2

Теперь складываем числа одним и тем же подкоренным выражением. Нельзя складывать или вычитать выражения с разными подкоренными выражениями. Сложение корней требует соблюдение этого правила.

Ответ получаем следующий:

√2+12√3-12√3+8√2=9√2

√2=1×√2 — надеюсь, то, что в алгебре принято опускать подобные элементы, не станет для вас новостью.

Выражения могут быть представлены не только квадратным корнем, но так же и с кубическим или корнем n-ной степени.

Сложение и вычитание корней с разными показателями степени, но с равнозначным подкоренным выражением, происходит следующим образом:

Если мы имеем выражение вида √a+∛b+∜b, то мы можем упростить это выражение так:

∛b+∜b=12×√b4 +12×√b3

12√b4 +12×√b3=12×√b4 + b3

Мы привели два подобных члена к общему показателю корня. Здесь использовалось свойство корней, которое гласит: если число степени подкоренного выражения и число показателя корня умножить на одно и то же число, то его вычисление останется неизменным.

На заметку: показатели степени складываются только при умножении.

Рассмотрим пример, когда в выражении присутствуют дроби.

5√8-4×√(1/4)+√72-4×√2

Будем решать по этапам:

5√8=5*2√2 — мы выносим из-под корня извлекаемую часть.

4√(1/4)=-4 √1/(√4)= — 4 *1/2= — 2

Если в тело корня представлено дробью, то часто этой дроби не измениться, если извлечь квадратный корень из делимого и делителя. В итоге мы получили описанное выше равенство.

√72-4√2=√(36×2)- 4√2=2√2

10√2+2√2-2=12√2-2

Вот и получился ответ.

Главное помнить, что из отрицательных чисел не извлекается корень с четным показателем степени. Если четной степени подкоренное выражение является отрицательным, то выражение является нерешаемым.

Сложение корней возможно только при совпадении подкоренных выражений, так как они являются подобными слагаемыми. То же самое относиться и к разности.

Сложение корней с разными числовыми показателями степени производиться посредством приведения к общей корневой степени обоих слагаемых. Это закон действует так же как приведение к общему знаменателю при сложении или вычитании дробей.

Если в подкоренном выражении имеется число, возведенное в степень, то это выражение можно упростить при условии, что между показателем корня и степени существует общий знаменатель.

В математике корни могут быть квадратными, кубическими или иметь любой другой показатель (степень), который пишется слева над знаком корня. Выражение, стоящее под знаком корня, называется подкоренным выражением. Сложение корней похоже на сложение членов алгебраического выражения, то есть требует определения подобных корней.

Шаги

Часть 1 из 2: Определение корней

Обозначение корней. Выражение под знаком корня () означает, что из этого выражения необходимо извлечь корень определенной степени.

  • Корень обозначают знаком.
  • Показатель (степень) корня пишется слева над знаком корня. Например, кубический корень из 27 записывается так: (27)
  • Если показатель (степень) корня отсутствует, то показатель считается равным 2, то есть это квадратный корень (или корень второй степени).
  • Число, записанное перед знаком корня, называется множителем (то есть это число умножается на корень), например 5 (2)
  • Если множителя перед корнем нет, то он равен 1 (напомним, что любое число, умноженное на 1, равняется самому себе).
  • Если вы впервые работаете с корнями, сделайте соответствующие пометки над множителем и показателем корня, чтобы не запутаться и лучше понять их назначение.

Запомните, какие корни можно складывать, а какие нельзя. Так же, как нельзя складывать разные члены выражения, например, 2а + 2b 4ab, вы не можете складывать разные корни.

  • Нельзя складывать корни с разными подкоренными выражениями, например, (2) + (3) (5). Но вы можете сложить числа, стоящие под одним корнем, например, (2 + 3) = (5) (квадратный корень из 2 примерно равен 1,414, квадратный корень из 3 примерно равен 1,732, а квадратный корень из 5 примерно равен 2,236).
  • Нельзя складывать корни с одинаковыми подкоренными выражениями, но разными показателями, например, (64) + (64) (эта сумма не равна (64), так как квадратный корень из 64 равен 8, кубический корень из 64 равен 4, 8 + 4 = 12, что гораздо больше, чем корень пятой степени из 64, который примерно равен 2,297).
  • Часть 2 из 2: Упрощение и сложение корней

    Определите и сгруппируйте подобные корни. Подобные корни – корни, у которых одинаковые показатели и одинаковые подкоренные выражения. Например, рассмотрим выражение:
    2 (3) + (81) + 2 (50) + (32) + 6 (3)

    • Во-первых, перепишите выражение так, чтобы корни с одинаковым показателем располагались последовательно.
      2 (3) + 2 (50) + (32) + 6 (3) + (81)
    • Затем перепишите выражение так, чтобы корни с одинаковым показателем и с одинаковым подкоренным выражением располагались последовательно.
      2 (50) + (32) + 2 (3) + 6 (3) + (81)

    Упростите корни. Для этого разложите (где возможно) подкоренные выражения на два множителя, один из которых вынесите из-под корня. В этом случае вынесенное число и множитель корня перемножаются.

  • В приведенном выше примере разложите число 50 на 2*25, а число 32 – на 2*16. Из 25 и 16 можно извлечь квадратные корни (соответственно 5 и 4) и вынести 5 и 4 из-под корня, соответственно умножив их на множители 2 и 1. Таким образом, вы получите упрощенное выражение: 10 (2) + 4 (2) + 2 (3) + 6 (3) + (81)
  • Число 81 можно разложить на множители 3*27, а из числа 27 можно извлечь кубический корень, равный 3. Это число 3 можно вынести из-под корня. Таким образом, вы получите еще более упрощенное выражение: 10 (2) + 4 (2) + 2 (3)+ 6 (3) + 3 (3)
  • Сложите множители подобных корней. В нашем примере есть подобные квадратные корни из 2 (их можно сложить) и подобные квадратные корни из 3 (их тоже можно сложить). У кубического корня из 3 подобных корней нет.

  • 10 (2) + 4 (2) = 14 (2).
  • 2 (3)+ 6 (3) = 8 (3).
  • Окончательное упрощенное выражение: 14 (2) + 8 (3) + 3 (3)
    • Не существует общепринятых правил порядка записи корней в выражении. Потому вы можете записывать корни в порядке возрастания их показателей и в порядке возрастания подкоренных выражений.

    Внимание, только СЕГОДНЯ!

    Все интересное

    Число, которое находится под знаком корня, часто мешает решению уравнения, с ним неудобно работать. Даже если оно возведено в степень, дробно или не может быть представлено в виде целого числа в определенной степени, можно попытаться вывести его из…

    Корнем из числа x называется такое число, которое при возведении в степень корня будет равно x. Множителем называется умножаемое число. То есть, в выражении вида x*&ordf-&radic-y нужно внести x под корень. Инструкция 1Определите степень…

    Если подкоренное выражение содержит набор математических действий с переменными, то иногда в результате его упрощения есть возможность получить относительно простое значение, часть которого можно вынести из под корня. Бывает полезно такое упрощение…

    Арифметические действия с корнями различной степени могут значительно упростить расчеты в физике и технике и сделать их более точными. При умножении и делении удобнее не извлекать корень из каждого сомножителя или делимого и делителя, а сначала…

    Квадратным корнем из числа x называют число a, которое при умножении само на себя дает число x: a * a = a^2 = x, x = a. Как и над любыми числами, над квадратными корнями можно выполнять арифметические операции сложения и вычитания. Инструкция …

    Корень в математике может иметь два значения: это арифметическое действие и каждое из решений уравнения, алгебраического, параметрического, дифференциального или любого другого. Инструкция 1Корень n-ной степени из числа a — это такое число, что…

    При выполнении различных арифметических действий с корнями часто бывает необходимо умение преобразовывать подкоренные выражения. Для упрощения расчетов может понадобиться вынести множитель за знак радикала или внести под него. Это действие можно…

    Корнем называют значок, обозначающий математическую операцию нахождения такого числа, возведение которого в указанную перед знаком корня степень должно дать число, указанное под этим самым знаком. Часто для решения задач, в которых присутствуют…

    Знаком корня в математических науках называется условное обозначение для корней. Число, находящееся под знаком корня, называется подкоренным выражением. n = a. Корни нечетной степени существуют для отрицательных и положительных чисел, а корни четной степени — только для положительных.…

    Приветствую, котаны! В прошлый раз мы подробно разобрали, что такое корни (если не помните, рекомендую почитать). Главный вывод того урока: существует лишь одно универсальное определение корней, которое вам и нужно знать. Остальное — брехня и пустая трата времени.

    Сегодня мы идём дальше. Будем учиться умножать корни, изучим некоторые проблемы, связанные с умножением (если эти проблемы не решить, то на экзамене они могут стать фатальными) и как следует потренируемся. Поэтому запасайтесь попкорном, устраивайтесь поудобнее — и мы начинаем.:)

    Вы ведь тоже ещё не вкурили?

    Урок получился довольно большим, поэтому я разделил его на две части:

    1. Сначала мы разберём правила умножения. Кэп как бы намекает: это когда есть два корня, между ними стоит знак «умножить» — и мы хотим что-то с этим сделать.
    2. Затем разберём обратную ситуацию: есть один большой корень, а нам приспичило представить его в виде произведения двух корней попроще. С какого перепугу это бывает нужно — вопрос отдельный. Мы разберём лишь алгоритм.

    Тем, кому не терпится сразу перейти ко второй части — милости прошу. С остальными начнём по порядку.

    Основное правило умножения

    Начнём с самого простого — классических квадратных корней. Тех самых, которые обозначаются $\sqrt{a}$ и $\sqrt{b}$. Для них всё вообще очевидно:

    Правило умножения. Чтобы умножить один квадратный корень на другой, нужно просто перемножить их подкоренные выражения, а результат записать под общим радикалом:

    \[\sqrt{a}\cdot \sqrt{b}=\sqrt{a\cdot b}\]

    Никаких дополнительных ограничений на числа, стоящие справа или слева, не накладывается: если корни-множители существуют, то и произведение тоже существует.

    Примеры. Рассмотрим сразу четыре примера с числами:

    \[\begin{align} & \sqrt{25}\cdot \sqrt{4}=\sqrt{25\cdot 4}=\sqrt{100}=10; \\ & \sqrt{32}\cdot \sqrt{2}=\sqrt{32\cdot 2}=\sqrt{64}=8; \\ & \sqrt{54}\cdot \sqrt{6}=\sqrt{54\cdot 6}=\sqrt{324}=18; \\ & \sqrt{\frac{3}{17}}\cdot \sqrt{\frac{17}{27}}=\sqrt{\frac{3}{17}\cdot \frac{17}{27}}=\sqrt{\frac{1}{9}}=\frac{1}{3}. \\ \end{align}\]

    Как видите, основной смысл этого правила — упрощение иррациональных выражений. И если в первом примере мы бы и сами извлекли корни из 25 и 4 без всяких новых правил, то дальше начинается жесть: $\sqrt{32}$ и $\sqrt{2}$ сами по себе не считаются, но их произведение оказывается точным квадратом, поэтому корень из него равен рациональному числу .

    Отдельно хотел бы отметить последнюю строчку. Там оба подкоренных выражения представляют собой дроби. Благодаря произведению многие множители сокращаются, а всё выражение превращается в адекватное число.

    Конечно, не всегда всё будет так красиво. Иногда под корнями будет стоять полная лажа — непонятно, что с ней делать и как преобразовывать после умножения. Чуть позже, когда начнёте изучать иррациональные уравнения и неравенства, там вообще будут всякие переменные и функции. И очень часто составители задач как раз и рассчитывают на то, что вы обнаружите какие-то сокращающиеся слагаемые или множители, после чего задача многократно упростится.

    Кроме того, совсем необязательно перемножать именно два корня. Можно умножить сразу три, четыре — да хоть десять! Правило от этого не поменяется. Взгляните:

    \[\begin{align} & \sqrt{2}\cdot \sqrt{3}\cdot \sqrt{6}=\sqrt{2\cdot 3\cdot 6}=\sqrt{36}=6; \\ & \sqrt{5}\cdot \sqrt{2}\cdot \sqrt{0,001}=\sqrt{5\cdot 2\cdot 0,001}= \\ & =\sqrt{10\cdot \frac{1}{1000}}=\sqrt{\frac{1}{100}}=\frac{1}{10}. \\ \end{align}\]

    И опять небольшое замечание по второму примеру. Как видите, в третьем множителе под корнем стоит десятичная дробь — в процессе вычислений мы заменяем её обычной, после чего всё легко сокращается. Так вот: очень рекомендую избавляться от десятичных дробей в любых иррациональных выражениях (т.е. содержащих хотя бы один значок радикала). В будущем это сэкономит вам кучу времени и нервов.

    Но это было лирическое отступление. Теперь рассмотрим более общий случай — когда в показателе корня стоит произвольное число $n$, а не только «классическая» двойка.{2n}}}=\left| a \right|. \\ \end{align}\]

    Подобные «махинации» могут здорово сэкономить вам время на экзамене или контрольной работе, поэтому запомните:

    Не спешите перемножать числа в подкоренном выражении. Сначала проверьте: вдруг там «зашифрована» точная степень какого-либо выражения?

    При всей очевидности этого замечания должен признать, что большинство неподготовленных учеников в упор не видят точные степени. Вместо этого они перемножают всё напролом, а затем удивляются: почему это получились такие зверские числа?:)

    Впрочем, всё это детский лепет по сравнению с тем, что мы изучим сейчас.

    Умножение корней с разными показателями

    Ну хорошо, теперь мы умеем перемножать корни с одинаковыми показателями. А что, если показатели разные? Скажем, как умножить обычный $\sqrt{2}$ на какую-нибудь хрень типа $\sqrt{23}$? Можно ли вообще это делать?

    Да конечно можно. Всё делается вот по этой формуле:

    Правило умножения корней.{2}}}=\sqrt{5}. \\ \end{align}\]

    Но тогда получается какая-то хрень:

    \[\sqrt{-5}=\sqrt{5}\]

    Этого не может быть, потому что $\sqrt{-5} \lt 0$, а $\sqrt{5} \gt 0$. Значит, для чётных степеней и отрицательных чисел наша формула уже не работает. После чего у нас есть два варианта:

    1. Убиться об стену констатировать, что математика — это дурацкая наука, где «есть какие-то правила, но это неточно»;
    2. Ввести дополнительные ограничения, при которых формула станет рабочей на 100%.

    В первом варианте нам придётся постоянно вылавливать «неработающие» случаи — это трудно, долго и вообще фу. Поэтому математики предпочли второй вариант.:)

    Но не переживайте! На практике это ограничение никак не влияет на вычисления, потому что все описанные проблемы касаются лишь корней нечётной степени, а из них можно выносить минусы.

    Поэтому сформулируем ещё одно правило, которое распространяется вообще на все действия с корнями:

    Прежде чем перемножать корни, сделайте так, чтобы подкоренные выражения были неотрицательны.2 = x, ?x = a. Как и над всякими числами, над квадратными корнями дозволено исполнять арифметические операции сложения и вычитания.

    Инструкция

    1. Во-первых, при сложении квадратных корней испробуйте извлечь эти корни. Это будет допустимо, если числа под знаком корня являются полными квадратами. Скажем, пускай задано выражение?4 + ?9. Первое число 4 – это квадрат числа 2. Второе число 9 – это квадрат числа 3. Таким образом получается, что: ?4 + ?9 = 2 + 3 = 5.

    2. Если под знаком корня нет полных квадратов, то испробуйте перенести из под знака корня множитель числа. Скажем, пускай дано выражение?24 + ?54. Разложите числа на множители: 24 = 2 * 2 * 2 * 3, 54 = 2 * 3 * 3 * 3. В числе 24 имеется множитель 4, тот, что дозволено перенести из под знака квадратного корня. В числе 54 – множитель 9. Таким образом, получается что: ?24 + ?54 = ?(4 * 6) + ?(9 * 6) = 2 * ?6 + 3 * ?6 = 5 * ?6. В данном примере в итоге выноса множителя из под знака корня получилось упростить заданное выражение.

    3. Пускай сумма 2-х квадратных корней является знаменателем дроби, скажем, A / (?a + ?b). И пускай перед вами стоит задача «избавиться от иррациональности в знаменателе». Тогда дозволено воспользоваться дальнейшим методом. Умножьте числитель и знаменатель дроби на выражение?a – ?b. Таким образом в знаменателе получится формула сокращенного умножения: (?a + ?b) * (?a – ?b) = a – b. По аналогии, если в знаменателе дана разность корней: ?a – ?b, то числитель и знаменатель дроби нужно умножить на выражение?a + ?b. Для примера, пускай дана дробь 4 / (?3 + ?5) = 4 * (?3 – ?5) / ((?3 + ?5) * (?3 – ?5)) = 4 * (?3 – ?5) / (-2) = 2 * (?5 – ?3).

    4. Разглядите больше непростой пример избавления от иррациональности в знаменателе. Пускай дана дробь 12 / (?2 + ?3 + ?5). Нужно умножить числитель и знаменатель дроби на выражение?2 + ?3 – ?5:12 / (?2 + ?3 + ?5) = 12 * (?2 + ?3 – ?5) / ((?2 + ?3 + ?5) * (?2 + ?3 – ?5)) = 12 * (?2 + ?3 – ?5) / (2 * ?6) = ?6 * (?2 + ?3 – ?5) = 2 * ?3 + 3 * ?2 – ?30.

    5. И наконец, если вам нужно только примерное значение, то дозволено посчитать значения квадратных корней на калькуляторе. Вычислите значения отдельно для всего числа и запишите с нужной точностью (скажем, два знака позже запятой). А после этого совершите требуемые арифметические операции, как с обыкновенными числами. Скажем, пускай нужно узнать примерное значение выражения?7 + ?5 ? 2,65 + 2,24 = 4,89.

    Видео по теме

    Обратите внимание!
    Квадратные корни ни в коем случае невозможно складывать как примитивные числа, т.е. ?3 + ?2 ? ?5!!!

    Полезный совет
    Если вы раскладываете число на множители, дабы перенести квадрат из под знака корня, то совершите обратную проверку – перемножьте все получившиеся множители и получите изначальное число.

    Сложение и вычитание квадратных корней примеры. Как складывать квадратные корни

    Содержимое:

    Складывать и вычитать квадратные корни можно только при условии, что у них одинаковое подкоренное выражение, то есть вы можете сложить или вычесть 2√3 и 4√3, но не 2√3 и 2√5. Вы можете упростить подкоренное выражение, чтобы привести их к корням с одинаковыми подкоренными выражениями (а затем сложить или вычесть их).

    Шаги

    Часть 1 Постигаем основы

    1. 1 (выражение под знаком корня). Для этого разложите подкоренное число на два множителя, один из которых является квадратным числом (число, из которого можно извлечь целый корень, например, 25 или 9). После этого извлеките корень из квадратного числа и запишите найденное значение перед знаком корня (под знаком корня останется второй множитель). Например, 6√50 — 2√8 + 5√12. Числа, стоящее перед знаком корня, являются множителями соответствующих корней, а числа под знаком корня – это подкоренные числа (выражения). Вот как решать данную задачу:
      • 6√50 = 6√(25 x 2) = (6 x 5)√2 = 30√2. Здесь вы раскладываете 50 на множители 25 и 2; затем из 25 извлекаете корень, равный 5, и 5 выносите из-под корня. Затем 5 умножаете на 6 (множитель у корня) и получаете 30√2.
      • 2√8 = 2√(4 x 2) = (2 x 2)√2 = 4√2. Здесь вы раскладываете 8 на множители 4 и 2; затем из 4 извлекаете корень, равный 2, и 2 выносите из-под корня. Затем 2 умножаете на 2 (множитель у корня) и получаете 4√2.
      • 5√12 = 5√(4 x 3) = (5 x 2)√3 = 10√3. Здесь вы раскладываете 12 на множители 4 и 3; затем из 4 извлекаете корень, равный 2, и 2 выносите из-под корня. Затем 2 умножаете на 5 (множитель у корня) и получаете 10√3.
    2. 2 Подчеркните корни, подкоренные выражения которых одинаковы. В нашем примере упрощенное выражение имеет вид: 30√2 — 4√2 + 10√3. В нем вы должны подчеркнуть первый и второй члены (30√2 и 4√2 ), так как у них одинаковое подкоренное число 2. Только такие корни вы можете складывать и вычитать.
    3. 3 Если вам дано выражение с большим количеством членов, многие из которых имеют одинаковые подкоренные выражения, используйте одинарное, двойное, тройное подчеркивание для обозначения таких членов, чтобы облегчить решение этого выражения.
    4. 4 У корней, подкоренные выражения которых одинаковы, сложите или вычтите множители, стоящие перед знаком корня, а подкоренное выражение оставьте прежним (не складывайте и не вычитайте подкоренные числа! ). Идея в том, чтобы показать, сколько всего корней с определенным подкоренным выражением содержится в данном выражении.
      • 30√2 — 4√2 + 10√3 =
      • (30 — 4)√2 + 10√3 =
      • 26√2 + 10√3

    Часть 2 Практикуемся на примерах

    1. 1 Пример 1: √(45) + 4√5.
      • Упростите √(45). Разложите 45 на множители: √(45) = √(9 x 5).
      • Вынесите 3 из-под корня (√9 = 3): √(45) = 3√5.
      • Теперь сложите множители у корней: 3√5 + 4√5 = 7√5
    2. 2 Пример 2: 6√(40) — 3√(10) + √5.
      • Упростите 6√(40). Разложите 40 на множители: 6√(40) = 6√(4 x 10).
      • Вынесите 2 из-под корня (√4 = 2): 6√(40) = 6√(4 x 10) = (6 x 2)√10.
      • Перемножьте множители перед корнем и получите 12√10.
      • Теперь выражение можно записать в виде 12√10 — 3√(10) + √5. Так как у первых двух членов одинаковые подкоренные числа, вы можете вычесть второй член из первого, а первый оставить без изменений.
      • Вы получите: (12-3)√10 + √5 = 9√10 + √5.
    3. 3 Пример 3. 9√5 -2√3 — 4√5. Здесь ни одно из подкоренных выражений нельзя разложить на множители, поэтому упростить это выражение не получится. Вы можете вычесть третий член из первого (так как у них одинаковые подкоренные числа), а второй член оставить без изменений. Вы получите: (9-4)√5 -2√3 = 5√5 — 2√3.
    4. 4 Пример 4. √9 + √4 — 3√2.
      • √9 = √(3 х 3) = 3.
      • √4 = √(2 х 2) = 2.
      • Теперь вы можете просто сложить 3 + 2, чтобы получить 5.
      • Окончательный ответ: 5 — 3√2.
    5. 5 Пример 5. Решите выражение, содержащее корни и дроби. Вы можете складывать и вычислять только те дроби, у которых общий (одинаковый) знаменатель. Дано выражение (√2)/4 + (√2)/2.
      • Найдите наименьший общий знаменатель этих дробей. Это число, которое делится нацело на каждый знаменатель. В нашем примере на 4 и на 2 делится число 4.
      • Теперь вторую дробь умножьте на 2/2 (чтобы привести ее к общему знаменателю; первая дробь уже приведена к нему): (√2)/2 х 2/2 = (2√2)/4.
      • Сложите числители дробей, а знаменатель оставьте прежним: (√2)/4 + (2√2)/4 = (3√2)/4 .
    • Перед суммированием или вычитанием корней обязательно упростите (если возможно) подкоренные выражения.

    Предупреждения

    • Никогда не суммируйте и не вычитайте корни с разными подкоренными выражениями.
    • Никогда не суммируйте и не вычитайте целое число и корень, например, 3 + (2x) 1/2 .
      • Примечание: «х» в одной второй степени и квадратный корень из «х» – это одно и то же (то есть x 1/2 = √х).

    Формулы корней. Свойства квадратных корней.

    Внимание!
    К этой теме имеются дополнительные
    материалы в Особом разделе 555.
    Для тех, кто сильно «не очень…»
    И для тех, кто «очень даже…»)

    В предыдущем уроке мы разобрались, что такое квадратный корень . Пришла пора разобраться, какие существуют формулы для корней , каковы свойства корней , и что со всем этим можно делать.2 = x, √x = a. Как и над любыми числами, над квадратными корнями можно выполнять арифметические операции сложения и вычитания.

    Инструкция

    • Во-первых, при сложении квадратных корней попробуйте извлечь эти корни. Это будет возможно, если числа под знаком корня являются полными квадратами. Например, пусть задано выражение √4 + √9. Первое число 4 – это квадрат числа 2. Второе число 9 – это квадрат числа 3. Таким образом получается, что: √4 + √9 = 2 + 3 = 5.
    • Если под знаком корня нет полных квадратов, то попробуйте вынести из под знака корня множитель числа. Например, пусть дано выражение √24 + √54. Разложите числа на множители: 24 = 2 * 2 * 2 * 3, 54 = 2 * 3 * 3 * 3. В числе 24 имеется множитель 4, который можно вынести из под знака квадратного корня. В числе 54 — множитель 9. Таким образом, получается что: √24 + √54 = √(4 * 6) + √(9 * 6) = 2 * √6 + 3 * √6 = 5 * √6. В данном примере в результате выноса множителя из под знака корня получилось упростить заданное выражение.
    • Пусть сумма двух квадратных корней является знаменателем дроби, например, A / (√a + √b). И пусть перед вами стоит задача «избавиться от иррациональности в знаменателе». Тогда можно воспользоваться следующим способом. Умножьте числитель и знаменатель дроби на выражение √a — √b. Таким образом в знаменателе получится формула сокращенного умножения: (√a + √b) * (√a — √b) = a – b. По аналогии, если в знаменателе дана разность корней: √a — √b, то числитель и знаменатель дроби необходимо умножить на выражение √a + √b. Для примера, пусть дана дробь 4 / (√3 + √5) = 4 * (√3 — √5) / ((√3 + √5) * (√3 — √5)) = 4 * (√3 — √5) / (-2) = 2 * (√5 — √3).
    • Рассмотрите более сложный пример избавления от иррациональности в знаменателе. Пусть дана дробь 12 / (√2 + √3 + √5). Необходимо умножить числитель и знаменатель дроби на выражение √2 + √3 — √5:
      12 / (√2 + √3 + √5) = 12 * (√2 + √3 — √5) / ((√2 + √3 + √5) * (√2 + √3 — √5)) = 12 * (√2 + √3 — √5) / (2 * √6) = √6 * (√2 + √3 — √5) = 2 * √3 + 3 * √2 — √30.
    • И наконец, если вам необходимо только приблизительное значение, то можно посчитать значения квадратных корней на калькуляторе. Вычислите значения отдельно для каждого числа и запишите с необходимой точностью (например, два знака после запятой). А затем совершите требуемые арифметические операции, как с обычными числами. Например, пусть необходимо узнать приблизительное значение выражения √7 + √5 ≈ 2,65 + 2,24 = 4,89.

    Свойства квадратных корней

    До сих пор мы осуществляли над числами пять арифметических операций: сложение, вычитание, умножение , деление и возведение в степень, причем при вычислениях активно использовали различные свойства этих операций, например а + b = b + а, аn-bn = (аb)n и т. д.

    В этой главе введена новая операция — извлечение квадратного корня из неотрицательного числа. Чтобы успешно ее использовать, нужно познакомиться со свойствами этой операции, что мы и сделаем в настоящем параграфе.

    Доказательство. Введем следующие обозначения:https://pandia.ru/text/78/290/images/image005_28.jpg» alt=»Равенство»Задание»> Имеются только свойства, касающиеся умножения и деления квадратных корней. Будьте внимательны и осторожны, не принимайте желаемое за действительное.

    Завершая параграф, отметим еще одно достаточно простое и в то же время важное свойство:
    если a > 0 и n — натуральное число , то

    Преобразование выражений, содержащих операцию извлечения квадратного корня

    До сих пор мы с вами выполняли преобразования толькорациональных выражений , используя для этого правила действий над многочленами и алгебраическими дробями, формулы сокращенного умножения и т. д. В этой главе мы ввели новую операцию — операцию извлечения квадратного корня; мы установили, что

    где, напомним, a, b — неотрицательные числа.

    Используя эти формулы , можно выполнять различные преобразования выражений, содержащих операцию извлечения квадратного корня. Рассмотрим несколько примеров, причем во всех примерах будем предполагать, что переменные принимают только неотрицательные значения.

    Пример 3. Внести множитель под знак квадратного корня:

    Пример 6 . Упростить выражение Решение. Выполним последовательные преобразования:

    Соблюдение Вашей конфиденциальности важно для нас. По этой причине, мы разработали Политику Конфиденциальности, которая описывает, как мы используем и храним Вашу информацию. Пожалуйста, ознакомьтесь с нашими правилами соблюдения конфиденциальности и сообщите нам, если у вас возникнут какие-либо вопросы.

    Сбор и использование персональной информации

    Под персональной информацией понимаются данные, которые могут быть использованы для идентификации определенного лица либо связи с ним.

    От вас может быть запрошено предоставление вашей персональной информации в любой момент, когда вы связываетесь с нами.

    Ниже приведены некоторые примеры типов персональной информации, которую мы можем собирать, и как мы можем использовать такую информацию.

    Какую персональную информацию мы собираем:

    • Когда вы оставляете заявку на сайте, мы можем собирать различную информацию, включая ваши имя, номер телефона, адрес электронной почты и т.д.

    Как мы используем вашу персональную информацию:

    • Собираемая нами персональная информация позволяет нам связываться с вами и сообщать об уникальных предложениях, акциях и других мероприятиях и ближайших событиях.
    • Время от времени, мы можем использовать вашу персональную информацию для отправки важных уведомлений и сообщений.
    • Мы также можем использовать персональную информацию для внутренних целей, таких как проведения аудита, анализа данных и различных исследований в целях улучшения услуг предоставляемых нами и предоставления Вам рекомендаций относительно наших услуг.
    • Если вы принимаете участие в розыгрыше призов, конкурсе или сходном стимулирующем мероприятии, мы можем использовать предоставляемую вами информацию для управления такими программами.

    Раскрытие информации третьим лицам

    Мы не раскрываем полученную от Вас информацию третьим лицам.

    Исключения:

    • В случае если необходимо — в соответствии с законом, судебным порядком, в судебном разбирательстве, и/или на основании публичных запросов или запросов от государственных органов на территории РФ — раскрыть вашу персональную информацию. Мы также можем раскрывать информацию о вас если мы определим, что такое раскрытие необходимо или уместно в целях безопасности, поддержания правопорядка, или иных общественно важных случаях.
    • В случае реорганизации, слияния или продажи мы можем передать собираемую нами персональную информацию соответствующему третьему лицу – правопреемнику.

    Защита персональной информации

    Мы предпринимаем меры предосторожности — включая административные, технические и физические — для защиты вашей персональной информации от утраты, кражи, и недобросовестного использования, а также от несанкционированного доступа, раскрытия, изменения и уничтожения.

    Соблюдение вашей конфиденциальности на уровне компании

    Для того чтобы убедиться, что ваша персональная информация находится в безопасности, мы доводим нормы соблюдения конфиденциальности и безопасности до наших сотрудников, и строго следим за исполнением мер соблюдения конфиденциальности.

    Упрощение выражений с квадратными корнями

    Результаты обучения

    • Упростите выражения с квадратными корнями, используя порядок операций
    • Упростите выражения с квадратными корнями, которые содержат переменные

    Квадратные корни и порядок операций

    При использовании порядка операций для упрощения выражения, имеющего квадратные корни, мы рассматриваем знак корня как символ группировки. Перед выполнением других операций мы упрощаем любые выражения под знаком радикала.

    пример

    Упростите: ⓐ [латекс] \ sqrt {25} + \ sqrt {144} [/ latex] ⓑ [латекс] \ sqrt {25 + 144} [/ latex].

    Решение

    ⓐ Используйте порядок операций.
    [латекс] \ sqrt {25} + \ sqrt {144} [/ латекс]
    Упростим каждый радикал. [латекс] 5 + 12 [/ латекс]
    Доп. [латекс] 17 [/ латекс]
    ⓑ Используйте порядок операций.
    [латекс] \ sqrt {25 + 144} [/ латекс]
    Добавить под радикальным знаком. [латекс] \ sqrt {169} [/ латекс]
    Упростить. [латекс] 13 [/ латекс]

    Обратите внимание на разные ответы в частях ⓐ и ⓑ приведенного выше примера. Важно правильно соблюдать порядок действий. В ⓐ мы сначала взяли каждый квадратный корень, а затем сложили его. В ⓑ мы сначала добавили под знаком корня, а затем нашли квадратный корень.{2} [/ латекс] [латекс] 6xy [/ латекс]

    упрощающих радикалов

    упрощающих радикалов Чтобы упростить радикалы, вместо того, чтобы искать точные квадраты или идеальные кубы внутри числа или переменной, как это показано в большинстве книг, я решаю задачи по-другому, и вот как.

    Вот шаги, необходимые для упрощения радикалов:

    Шаг 1 : Найдите разложение числа внутри корня на простые множители.Начните с деления числа на первое простое число 2 и продолжайте деление на 2, пока не получите десятичную дробь или остаток. Затем разделите на 3, 5, 7 и т. Д., Пока не останутся только простые числа. Щелкните ссылку, чтобы увидеть несколько примеров первичной факторизации. Также учитывайте любые переменные внутри радикала.
    Шаг 2 : Определите индекс радикала. Индекс сообщает вам, сколько чисел вам нужно собрать, чтобы иметь возможность переместить это число или переменную из радикала во вне радикала.Например, если индекс равен 2 (квадратный корень), вам нужно два одинаковых вида, чтобы переместиться изнутри радикала в сторону вне радикала. Если индекс равен 3 (кубический корень), то вам нужны тройки одного сорта, чтобы переместиться изнутри корня во вне корня.
    Шаг 3 : Переместите каждую группу чисел или переменных из радикала в радикал. Если чисел или переменных недостаточно, чтобы создать группу из двух, трех или чего-то еще, оставьте эти числа или переменные внутри радикала.Обратите внимание, что каждая группа чисел или переменных записывается один раз, когда они выходят за пределы радикала, потому что теперь они составляют одну группу.
    Шаг 4 : Упростите выражения как внутри, так и вне радикала, умножая. Умножьте все числа и переменные внутри радикала вместе. Умножьте все числа и переменные вне корня вместе.

    Пример 1 — Упростить:

    Шаг 1 : Найдите разложение числа внутри корня на простые множители.
    Шаг 2 : Определите индекс радикала. В этом случае индекс равен двум, потому что это квадратный корень, а это значит, что нам нужны два одинаковых типа.
    Шаг 3 : Переместите каждую группу чисел или переменных из радикала за пределы радикала. В этом случае пара двойки и тройки вышла за пределы радикала.
    Шаг 4 : Упростите выражения как внутри, так и вне корня путем умножения.

    Пример 2 — Упростить:

    Шаг 1 : Найдите разложение числа внутри корня на простые множители и разложите на множители каждую переменную внутри корня.
    Шаг 2 : Определите индекс радикала. В данном случае индекс равен трем, потому что это кубический корень, а это значит, что нам нужны три одинаковых.
    Шаг 3 : Переместите каждую группу чисел или переменных из радикала в радикал.В этом случае тройки, x (две группы) и y вышли за пределы радикала.
    Шаг 4 : Упростите выражения как внутри, так и вне корня путем умножения.

    Нажмите здесь, чтобы узнать о практических задачах

    Пример 3 — Упростить:

    Шаг 1 : Найдите разложение числа внутри корня на простые множители и разложите на множители каждую переменную внутри корня.
    Шаг 2 : Определите индекс радикала. В этом случае индекс равен пяти, потому что это корень пятой степени, а это значит, что нам нужны пять одинаковых чисел.
    Шаг 3 : Переместите каждую группу чисел или переменных из радикала в радикал. В этом случае тройки, x и y выходят за пределы радикала.
    Шаг 4 : Упростите выражения как внутри, так и вне корня путем умножения.

    Нажмите здесь, чтобы узнать о практических задачах

    Пример 4 — Упростить:

    Шаг 1 : Найдите разложение числа внутри корня на простые множители и разложите на множители каждую переменную внутри корня.
    Шаг 2 : Определите индекс радикала. В этом случае индекс равен двум, потому что это квадратный корень, а это значит, что нам нужны два одинаковых типа.
    Шаг 3 : Переместите каждую группу чисел или переменных из радикала в радикал. В этом случае двойки, тройки, x (две группы) и y (четыре группы) вышли за пределы радикала.
    Шаг 4 : Упростите выражения как внутри, так и вне корня путем умножения.

    Нажмите здесь, чтобы узнать о практических задачах

    Пример 5 — Упростить:

    Шаг 1 : Найдите разложение числа внутри корня на простые множители и разложите на множители каждую переменную внутри корня.
    Шаг 2 : Определите индекс радикала. В данном случае индекс равен четырем, потому что это корень четвертой степени, а это значит, что нам нужны четыре одинаковых числа.
    Шаг 3 : Переместите каждую группу чисел или переменных из радикала за пределы радикала. В этом случае пара двоек и y вышла за пределы радикала.
    Шаг 4 : Упростите выражения как внутри, так и вне корня путем умножения.

    Нажмите здесь, чтобы узнать о практических задачах

    Пример 6 — Упростить:

    Шаг 1 : Найдите разложение числа внутри корня на простые множители и разложите на множители каждую переменную внутри корня.
    Шаг 2 : Определите индекс радикала. В этом случае индекс равен трем, потому что это кубический корень, а это значит, что нам нужны три одинаковых.
    Шаг 3 : Переместите каждую группу чисел или переменных из радикала в радикал. В этом случае двойки, тройки, х и у (две группы) вышли за пределы радикала.
    Шаг 4 : Упростите выражения как внутри, так и вне корня путем умножения.

    Нажмите здесь, чтобы узнать о практических задачах

    Решение радикальных уравнений

    Решение радикальных уравнений

    A Радикальное уравнение — это уравнение, в котором переменная находится под радикалом.Чтобы решить радикальное уравнение:

    1. Выделите радикальное выражение, содержащее переменную. Если переменная включает более одного радикального выражения, выделите одно из них.

    2. Возвести обе части уравнения до индекса радикала.

    3. Если радикальное уравнение все еще существует, повторите шаги 1 и 2; в противном случае решите полученное уравнение и проверьте ответ в исходном уравнении.

    Возведением обеих частей уравнения в степень могут быть введены некоторые решения, которые не делают исходное уравнение истинным. Эти решения называются посторонними решениями.

    Пример 1

    Решить.

    Выделите радикальное выражение.

    Поднимите обе стороны до индекса корня; в этом случае квадрат с обеих сторон.

    Это квадратное уравнение теперь можно решить либо факторизацией, либо применением формулы корней квадратного уравнения.

    Применяя квадратичную формулу,

    Теперь проверьте результаты.

    Если,

    Если x = –5,

    Решение: или x = –5.

    Пример 2

    Решить.

    Выделите радикальное выражение.

    Нет решения, так как не может иметь отрицательного значения.

    Пример 3

    Решить.

    Выделите одно из радикальных выражений.

    Поднимите обе стороны до индекса корня; в этом случае квадрат с обеих сторон.

    Это все еще радикальное уравнение. Выделите радикальное выражение.

    Поднимите обе стороны до индекса корня; в этом случае квадрат с обеих сторон.

    Эту проблему можно решить либо факторингом, либо применением формулы корней квадратного уравнения.

    Применяя квадратичную формулу,

    Проверьте решения.

    Если x = 10,

    Итак, x = 10 не является решением.

    Если x = 2,

    Единственное решение: x = 2.

    Пример 4

    Решить.

    Выделите радикал, содержащий переменную.

    Поскольку радикалы с нечетными индексами могут иметь отрицательные ответы, у этой проблемы есть решения. Поднимите обе части уравнения до индекса радикала; в этом случае кубик с обеих сторон.

    Проверка решения x = –15 предоставляется вам.

    Упрощение радикальных выражений

    Прежде чем вы сможете упростить радикальное выражение, вы должны знать важные свойства радикалов .

    ПРОДУКТ СОБСТВЕННОСТИ КВАДРАТНЫХ КОРНЕЙ

    Для всех реальных чисел а а также б ,

    а ⋅ б знак равно а ⋅ б

    То есть квадратный корень из произведения совпадает с произведением квадратных корней.

    Есть аналогичное свойство частного:

    Для всех реальных чисел а а также б , б ≠ 0 :

    а б знак равно а б

    Поскольку отрицательное число, умноженное на отрицательное, всегда является положительным числом, при извлечении квадратного корня необходимо помнить, что ответ будет как положительным, так и отрицательным числом или выражением. Например а ⋅ а знак равно а 2 , а также ( — а ) ⋅ ( — а ) знак равно а 2 .Такие двойственные ответы мы обычно будем обозначать как ± а .

    УПРОЩЕНИЕ РАДИКАЛА

    Идея здесь состоит в том, чтобы найти идеальный квадратный множитель подкоренного выражения, записать подкоренное выражение как произведение, а затем использовать свойство произведения для упрощения.

    Пример 1:

    Упрощать. 45

    9 идеальный квадрат, который также является фактором 45 .

    45 знак равно 9 ⋅ 5

    Используйте свойство продукта.

    9 ⋅ 5 знак равно 9 ⋅ 5 знак равно ± 3 5

    Если число под корнем не имеет точных квадратных множителей, то его нельзя упростить дальше.Например, число 17 нельзя упростить дальше, потому что единственные факторы 17 или же 17 а также 1 . Итак, нет никаких точных квадратных факторов, кроме 1 .

    Пример 2:

    Упрощать. 12 3

    Используйте свойство частного, чтобы писать под одним знаком квадратного корня.

    12 3 знак равно 12 3

    Делить.

    знак равно 4 знак равно ± 2

    Выражение считается упрощенным, только если в знаменателе отсутствует знак корня. Если у нас есть радикальный признак, мы должны рационализировать знаменатель . Это достигается умножением числителя и знаменателя на радикал в знаменателе. Обратите внимание, что здесь мы просто умножаем на специальную форму 1 , поэтому значение выражения не меняется.

    Пример 3:

    Упрощать. 5 6

    5 6 знак равно 5 6 ⋅ 6 6

    Упрощать.

    знак равно 30 6

    Иногда нам нужно использовать комбинацию шагов.

    Пример 4:

    Упрощать.21 год 9

    21 год а также 9 имеют общий фактор 3 , поэтому уменьшаем дробь под радикалом.

    21 год 9 знак равно 7 3 знак равно 7 3

    Теперь рационализируйте знаменатель.

    7 3 ⋅ 3 3 знак равно 21 год 3

    Мы можем добавить или вычесть два радикальных выражения только в том случае, если подкоренные выражения совпадают.Например, 17 + 13 не может быть далее упрощен. Но мы можем упростить 5 2 + 3 2 используя распределительное свойство , потому что подкоренные выражения такие же.

    5 2 + 3 2 знак равно ( 5 + 3 ) 2 знак равно 8 2

    Будь осторожен! Иногда подкоренные выражения выглядят по-разному, но можно упростить и получить то же подкоренное выражение.

    Пример 5:

    Упрощать. 50 + 32

    Упростите оба радикала:

    50 + 32 знак равно 25 ⋅ 2 + 16 ⋅ 2 знак равно ± 5 2 ± 4 2

    Теперь подкоренные выражения такие же.

    Итак, мы можем добавить, используя свойство дистрибутива.

    Упрощение радикальных выражений

    Мы знаем, что квадратный корень не является действительным числом, когда подкоренное выражение x отрицательно. Таким образом, мы заключаем, что область состоит из всех действительных чисел, больших или равных 0. Здесь мы выбираем 0 и некоторые положительные значения для x , вычисляем соответствующие значения y и строим полученные упорядоченные пары.

    После нанесения точек мы можем нарисовать график функции квадратного корня.

    Пример 11: Для функции f (x) = x + 2 найдите f (−2), f (2) и f (6).

    Решение: Замените x каждым из указанных значений.

    Поскольку корень куба может быть как отрицательным, так и положительным, мы заключаем, что домен состоит из всех действительных чисел. Для полноты картины выберите несколько положительных и отрицательных значений для x , а также 0, а затем вычислите соответствующие значения y .

    Постройте точки и нарисуйте график функции кубического корня.

    Пример 12: Для функции g (x) = x − 13 найдите g (−7), g (0) и g (55).

    Решение: Замените x каждым из указанных значений.

    Тематические упражнения

    Часть A: Упрощение радикальных выражений

    Упростить. ( Предположим, что все переменные представляют собой положительные числа. )

    1.36a2

    2. 121b2

    3. x2y2

    4. 25x2y2z2

    5. 180×3

    6. 150y3

    7. 49a3b2

    8. 4a4b3c

    9. 45x5y3

    10. 50x6y4

    11. 64р2с6т5

    12. 144р8с6т2

    13. (x + 1) 2

    14. (2х + 3) 2

    15.4 (3x − 1) 2

    16. 9 (2x + 3) 2

    17. 9x325y2

    18. 4x59y4

    19. m736n4

    20. 147м9н6

    21. 2р2с525т4

    22. 36р5с2т6

    23. 27a33

    24. 125b33

    25. 250x4y33

    26. 162a3b53

    27. 64x3y6z93

    28. 216x12y33

    29.8x3y43

    30. 27x5y33

    31. a4b5c63

    32. a7b5c33

    33. 8x427y33

    34. x5125y63

    35. 360р5с12т133

    36. 540р3с2т93

    37. 81×44

    38. x4y44

    39. 16x4y84

    40. 81x12y44

    41. a4b5c64

    42. 54a6c84

    43.128×64

    44. 243y74

    45. 32м10н55

    46. 37м9н105

    47. −34×2

    48. 79y2

    49. −5x4x2y

    50. −3y16x3y2

    51. 12aba5b3

    52. 6a2b9a7b2

    53. 2x⋅8×63

    54. −5×2⋅27×33

    55. 2ab⋅ − 8a4b53

    56.5a2b⋅ − 27a3b33

    Запишем следующее в виде радикального выражения с коэффициентом 1.

    57. 52x

    58. 23 года

    59. 2×3

    60. 3y2

    61. ab10a

    62. 2ab2a

    63. m2nmn

    64. 2м2н33н

    65. 5 2×3

    66. 3 5лет

    67.2x⋅33

    68. 3 года 23

    Предположим, что переменная может представлять любое действительное число, а затем упростить.

    69. 4×2

    70. 25y2

    71. 8y33

    72. 125a33

    73. 64×44

    74. 81y44

    75. 36a4

    76. 100a8

    77. 4a6

    78.a10

    79. 18a4b5

    80. 48a5b3

    81. 128x6y86

    82. a6b7c86

    Часть B: Формулы, включающие радикалы

    Перехваты y для любого графика будут иметь форму (0, y ), где y — действительное число. Следовательно, чтобы найти y -перехват, установите x = 0 и решите относительно y .Найдите интерцепторы y для следующего.

    83. y = x + 4−1

    84. y = x + 1−3

    85. y = x − 13 + 2

    86. y = x + 13−3

    Используйте формулу расстояния, чтобы вычислить расстояние между заданными двумя точками.

    87. (5, −7) и (3, −8)

    88. (−9, 7) и (−8, 4)

    89. (−3, −4) и (3, −6)

    90.(−5, −2) и (1, −6)

    91. (-1, 1) и (-4, 10)

    92. (8, −3) и (2, −12)

    Разложите на множители подкоренное выражение, а затем упростите. ( Предположим, что все выражения положительны. )

    93. x2−6x + 9

    94. x2−10x + 25

    95. 4×2 + 12x + 9

    96. 9×2 + 6x + 1

    97. Скорость транспортного средства до того, как были задействованы тормоза, можно оценить по длине следов заноса, оставленных на дороге.На сухом асфальте скорость v в милях в час можно оценить по формуле v = 5d, где d представляет собой длину следов заноса в футах. Оцените скорость транспортного средства перед тем, как задействовать тормоза на сухом асфальте, если оставленные следы заноса составляют 36 футов.

    98. Радиус r сферы можно рассчитать по формуле r = 6π2V32π, где V представляет объем сферы. Каков радиус сферы, если ее объем составляет 36π кубических сантиметров?

    Период T маятника в секундах определяется формулой

    Т = 2πL32

    , где L — длина в футах.Рассчитайте период, учитывая следующие значения длины. Укажите точное значение и округлите приблизительное значение с точностью до десятых долей секунды.

    99,8 футов

    100. 32 фута

    101. 1/2 фута

    102. 1/8 фута

    Время, t , в секундах, в течение которого объект находится в свободном падении, определяется формулой т = s4

    , где s представляет собой расстояние в футах, на которое он упал.Рассчитайте время, за которое объект упадет, учитывая следующие расстояния. Укажите точное значение и округлите приблизительное значение с точностью до десятых долей секунды.

    103. 48 футов

    104. 80 футов

    105,192 футов

    106. 288 футов

    Часть C: Радикальные функции

    Учитывая функцию, вычислите следующее.

    107.f (x) = x − 1, найти f (1), f (2) и f (5)

    108. f (x) = x + 5, найти f (−5), f (−1) и f (20)

    109. f (x) = x + 3, найти f (0), f (1) и f (16)

    110. f (x) = x − 5, найти f (0), f (1) и f (25)

    111. g (x) = x3, найти g (−1), g (0) и g (1)

    112. g (x) = x + 73, найти g (−15), g (−7) и g (20)

    113. g (x) = x3−2, найти g (−1), g (0) и g (8)

    114. g (x) = x − 13 + 2, найти g (0), g (2) и g (9)

    Для каждой функции заполните таблицу.

    115. f (x) = x + 1

    116. f (x) = x − 2

    117. f (x) = x3 + 1

    118. f (x) = x + 23

    Часть D: Обсуждение

    119. Задайте для x такое значение, чтобы x2 ≠ x. Объясните, почему важно предполагать, что переменные представляют собой положительные числа.

    120. Изучите и обсудите достижения Кристофа Рудольфа. За что ему приписывают?

    121. Изучите и обсудите методы, используемые для вычисления квадратных корней, до того, как электронные калькуляторы начнут широко использоваться.

    122. Что такое surd и откуда взялось это слово?

    Радикалы, силы и корни: Учебное пособие по решению уравнений

    Решение уравнений

    Решение уравнений, о возможности которых вы даже не догадывались

    Ну, хорошо, хорошо.Посмотрите, кто вернулся, чтобы получить больше. Так ты думаешь, что у тебя есть основы, не так ли? Вы думаете, что освоили упрощающие радикалы? А как насчет дробных и отрицательных показателей? Это может быть правдой, но вы еще не освоили эту главу, пока не освоите решение для отсутствующей переменной. Это действительно полный опыт «летать в руке», который вам абсолютно необходим.

    Хорошо это или плохо, но мы предполагаем, что у вас уже есть основы решения алгебраических уравнений. Мы ложимся спать ночью в надежде, что вы умеете складывать, вычитать, умножать и делить свой путь к решению для x .Однако одна вещь, которую вы, возможно, не видели раньше, — это то, как отменить квадратный или квадратный корень, чтобы получить маленькое x , все от его одинокого.

    Мы позволим себе пропустить несколько примеров задач. Мы думаем, что вы быстро освоитесь.

    А теперь иди лови мух.

    Пример задачи

    Решите для x .

    В конечном итоге наша цель в любой разрешающей ситуации — получить переменную сама по себе.Для этого мы хотим отменить все операции, выполненные с x . Первое, что нужно отменить, — это извлечение квадратного корня, поэтому мы воспользуемся обратной операцией извлечения квадратного корня. Другими словами, квадрат с обеих сторон.


    x + 2 = 9

    Это упрощает управление. Давайте продолжим и отменим сложение, вычтя 2 с обеих сторон.

    x + 2 = 9
    x = 7

    Fly номер один: проверьте.

    Пример задачи

    Решите для x .

    И снова перед нами стоит задача самостоятельно получить x . Мы начнем с упрощения этой безумной радикально экспоненциальной штуковины слева. Он также будет отвечать на свое другое название: термин. Для этого нам нужно взять корень третьей степени из (- x ) 3 . Поскольку это операции, обратные друг другу, мы имеем…

    Nice. Корень куба отменяет экспоненту. Отсюда это довольно простая алгебра. Мы закончим, добавив x и 2 с обеих сторон.

    x + 4 = 2 x — 2
    6 = 3 x

    Наконец, чтобы отменить умножение, мы можем разделить обе части на 3. Это дает нам окончательный ответ.

    6 = 3 x
    2 = x

    Пример задачи

    Решить:.

    Вот и поговорим. Нашим первым шагом должно быть упрощение этой вещи. Вопрос в том, как? Все, что мы делаем, это переписываем левую часть, используя дробные показатели. Для правой стороны мы будем использовать наши свойства экспоненты, но сохраним позитив.После этого мы оценим нашу ситуацию.

    Умножение обеих сторон на x здесь кажется правильным решением. Однако, поскольку это означает, что x больше не в знаменателе, важно отметить, что независимо от того, куда ведет нас наша работа, x не может равняться 0.

    x 1 + 3 / 2 = 1
    x 2/2 + 3/2 = 1
    x 5/2 = 1

    Уравнение, которое мы сейчас имеем, можно записать двумя способами: x 5/2 = 1 или.Мы любители вернуться к нефракционной версии, чтобы закончить. Это позволяет нам легко увидеть, что следующим шагом будет возведение обеих сторон в квадрат, чтобы избавиться от надоедливого квадратного корня.

    x 5/2 = 1


    x 5 = 1

    Наконец, мы можем отменить показатель степени, взяв корень пятой степени из обеих частей.

    x 5 = 1

    x = 1

    В целом, эта задача решена очень хорошо, поскольку 1 2 равно 1, а также просто 1.Жизнь не всегда может быть такой доброй.

    Пример задачи

    Решить:.

    Еще раз, нам нужно найти x . Однако у него есть серьезная математическая броня: существует масса различных операций, защищающих его от того, чтобы он существовал сам по себе. Время долбить их по одному. Для начала добавим по 3 с обеих сторон.

    Затем мы сокращаем кубические корни, создавая кубы с обеих сторон.

    x 2 + 2 = 27

    Чтобы отменить сложение 2, мы вычтем 2 с обеих сторон.В этом нет ничего удивительного.

    x 2 + 2 = 27
    x 2 = 25

    Наконец, мы можем извлечь квадратный корень из обеих частей, чтобы найти ответ. Но вот в чем дело: каждое положительное действительное число имеет два квадратных корня. Чуть позже мы рассмотрим, что такое действительное число, а сейчас скажем так:

    x 2 = 25

    x = ± 5

    Этот символ ± называется » знак плюс или минус «и просто означает, что у нас есть два решения: +5 или -5.

    Пример задачи

    Решить:.

    Эта задача выглядит достаточно простой. Верно? Поскольку здесь мы не можем комбинировать подобные термины, мы хотим избавиться от надоедливого квадратного корня. Для этого у нас нет другого выбора, кроме как возвести обе стороны в квадрат.



    ( x + 3) 2 = 16 x

    Отлично. Раньше все не выглядело слишком сложным, но теперь слева есть бином. Ой. По крайней мере, у нас не осталось квадратных корней. В этот момент молодые новички могут просто сидеть и смотреть на них.Но нужно что-то делать. Почему бы не перемножить двучлен?

    ( x + 3) 2 = 16 x
    ( x + 3) ( x + 3) = 16 x
    x
    2 + 6 x + 9 = 16 x

    Сейчас все еще не так радужно. Но согласитесь, мы приближаемся. В прошлый раз, когда у нас была квадратичная, лучший способ решить это приравнять значение 0. Та же идея применима и здесь. Вычтем 16 x с обеих сторон.

    x 2 + 6 x + 9 = 16 x
    x 2 — 10 x + 9 = 0

    Далее, если мы не сможем разложить это на множители, мы Придется вытащить квадратную формулу. Понимаете, что мы имеем в виду, говоря, что это секция ловли мухи? К счастью для нас, квадратичные множители очень хороши.

    x 2 — 10 x + 9 = 0
    ( x — 1) ( x — 9) = 0

    Наконец, мы знаем, что если два объекта имеют произведение 0, один из них просто должен быть 0.Выбора нет. Чтобы решить, мы установим каждый коэффициент равным 0. Это даст нам два решения:

    ( x — 9) = 0
    x = 9

    Или:

    ( x — 1) = 0
    x = 1

    Стрела. Жареный.

    Упрощение выражений с помощью более высоких корней

    Упрощение выражений с использованием более высоких корней

    До сих пор в этом руководстве мы работали с квадратами и квадратными корнями. Теперь мы расширим нашу работу, включив в нее высшие силы и высшие корни.{5} \ hfill & & \ phantom {\ rule {2em} {0ex}} n \ phantom {\ rule {0.2em} {0ex}} \ text {до пятого} \ hfill \ end {array} \)

    Термины «квадрат» и «куб» происходят от формул для площади квадрата и объема куба.

    Было бы полезно иметь таблицу степеней целых чисел от \ (- 5 \ phantom {\ rule {0.2em} {0ex}} \ text {до} \ phantom {\ rule {0.2em} {0ex }} 5 \). См. Рисунок ниже.

    От первой до пятой степени целых чисел от \ (- 5 \) до \ (5. \)

    Обратите внимание на знаки на рисунке ниже.{5} & \ hfill = & -32 \ hfill & & \ hfill \ phantom {\ rule {4em} {0ex}} \ sqrt [5] {- 32} & = \ hfill & -2 \ hfill \ end {массив } \)

    Можно ли получить четный корень отрицательного числа? Нет. Мы знаем, что квадратный корень отрицательного числа не является действительным числом. То же верно и для любого четного корня. Даже корни отрицательных чисел не являются действительными числами. Нечетные корни отрицательных чисел — действительные числа.

    Свойства \ (\ sqrt [n] {a} \)

    Если \ (n \) — четное число и

    • \ (a \ ge 0 \), тогда \ (\ sqrt [n] { a} \) является действительным числом
    • \ (a <0 \), тогда \ (\ sqrt [n] {a} \) не является действительным числом

    Когда \ (n \) является нечетным числом, \ (\ sqrt [n] {a} \) — действительное число для всех значений \ (a \).

    Добавить комментарий

    Ваш адрес email не будет опубликован. Обязательные поля помечены *